Category Archives: Lớp 9

ĐỀ THI THỬ VÀO LỚP 10 TRUNG TÂM STAR EDUCATION TOÁN CHUYÊN – 2020

Bài 1. (1,5 điểm )

a) Cho $f(x)=x^{2}-a x+a^{2}-4$, trong đó $a$ là tham số. Tìm giá trị của $a$, sao cho phương trình $f(x)=0$ có hai nghiệm thực $x_{1}$ và $x_{2}$ sao cho $\left|x_{1}^{3}-x_{2}^{3}\right| \leq 4$.

b) Giải phương trình: $\frac{1+3 \sqrt{x}}{4 x+\sqrt{2+x}}-1=0$.

Bài 2. (1,5 điểm ) Cho $x, y>0$ thỏa mãn $2 y>x$ và $11(\sqrt{x}+\sqrt{y})+4 \sqrt{x y}=26$

a) Tìm giá trị nhỏ nhất của biểu thức: $T=11(x+y)+\frac{1}{x}+\frac{1}{y}+2021$

b) Chứng minh rằng: $\frac{1}{x^{3}(2 y-x)}+x^{2}+y^{2} \geq 3$

Bài 3. (1,0 điểm) Cho hàm số bậc hai $f(x)=a x^{2}+b x+c,(a \neq 0)$. Biết rằng phương trình $f(x)=x$ vô nghiệm. Chứng minh rằng phương trình $f(f(x))=x$ cũng vô nghiệm.

Bài 4. $\left(1,5\right.$ điểm) Cho $x, y \in N$ thỏa mãn: $3^{x}+171=y^{2}$.

a) Chứng minh rằng: $x: 2$.

b) Tìm các cặp số $x, y$ thỏa mãn phương trình.

Bài 5. (3,0 điểm) Cho đường tròn $(O)$ và điểm $P$ nằm ngoài đường tròn. Vẽ các tiếp tuyến $P A, P B$ đến $(O)$ với $A, B$ là các tiếp điểm. $C$ là điểm trên cung nhỏ $A B$, tiếp tuyến tại $C$ cắt $P A, P B$ và $P O$ lần lượt tại $D, E, F$.

a) Gọi $H$ là giao điểm của đường tròn ngoại tiếp tam giác $P D E$ và $P O$, kéo dài $H C$ cắt đường tròn $P D E$ tại điểm $G$. Chứng minh rằng tứ giác $P F C G$ nội tiếp.

b) Gọi $I$ là tâm đường tròn nội tiếp tam giác $\triangle P D E$. Chứng minh rằng tứ giác $D O E I$ nội tiếp.

c) Chứng minh rằng $H$ là tâm đường tròn ngoại tiếp tam giác $\triangle D O E$.

d) Chứng minh rằng đường tròn ngoại tiếp các tam giác $P A B, P D E$ và $P C F$ cùng đi qua một điểm khác $P$.

Bài 6. (1,5 điểm) Trên mặt phẳng cho 17 điểm, trong đó không có ba điểm nào thẳng hàng. Qua hai điểm bất kì ta vẽ được một đoạn thẳng và trên đoạn thẳng đó ghi một số nguyên dương (các số ghi trên các đoạn thẳng khác nhau là các số nguyên dương khác nhau). Ta tô màu mỗi đoạn thẳng bằng một trong ba màu: đỏ, xanh và vàng.

a) Chứng minh rằng tồn tại một tam giác có ba cạnh cùng màu.

b) Chứng minh rằng tồn tại một tam giác có các cạnh là các đoạn thẳng đã vẽ và tổng các số ghi trên các cạnh của tam giác đó là hợp số.

LỜI GIẢI

 

Bài 1. a) Để phương trình có hai nghiệm thực $x_{1}$ và $x_{2}$ thì $\Delta=a^{2}-4\left(a^{2}-4\right)=16-3 a^{2} \geq 0$. Theo định lý Vietè ta có: $\left\{\begin{array}{l}x_{1}+x_{2}=a \\ x_{1} x_{2}=a^{2}-4\end{array}\right.$, do đó:

$\left|x_{1}^{3}-x_{2}^{3}\right|=\left|x_{1}-x_{2}\right|\left[\left(x_{1}+x_{2}\right)^{2}-x_{1} x_{2}\right]=\left|x_{1}-x_{2}\right|\left[a^{2}-a^{2}+4\right]=4\left|x_{1}-x_{2}\right| \leq 4$

Lại có:

$0 \leq\left|x_{1}-x_{2}\right|=\sqrt{\left(x_{1}+x_{2}\right)^{2}-4 x_{1} x_{2}}=\sqrt{a^{2}-4\left(a^{2}-4\right)}=\sqrt{16-3 a^{2}} \leq 1$

Vì vậy, ta có: $a \in\left[-\frac{4 \sqrt{3}}{3},-\sqrt{5}\right] \cup\left[\sqrt{5} ; \frac{4 \sqrt{3}}{3}\right]$.

b) $Đ K: x \geq 0$. Phương trình đã cho tương đương:

$1+3 \sqrt{x}-4 x-\sqrt{2+x}=0 $

$\Leftrightarrow 3 \sqrt{x}-\sqrt{2+x}=4 x-1 $

$\Leftrightarrow(8 x-2)=(4 x-1)(3 \sqrt{x}+\sqrt{2+x}) $

$\Leftrightarrow(4 x-1)[(3 \sqrt{x}+\sqrt{2+x})-2]=0 $

$\Leftrightarrow\left[\begin{array}{l}4 x-1=0 \\3 \sqrt{x}+\sqrt{2+x}=2\end{array}\right.$

Từ đó ta tính được hai nghiệm của phương trình là: $S=[\frac{1}{4} ; \frac{7-3 \sqrt{5}}{8}]$.

Bài 2. Áp dụng bất đẳng thức Cauchy ta có:

$11\left(\frac{x+y+2}{2}\right)+2(x+y) \geq 11 \sqrt{2(x+y)}+2(x+y) \geq 11(\sqrt{x}+\sqrt{y})+4 \sqrt{x y}=26$

Do đó: $\frac{15}{2}(x+y) \geq 15 \Leftrightarrow x+y \geq 2$

a) Áp dụng bất đẳng thức Cauchy ta có:

$T=11(x+y)+\frac{1}{x}+\frac{1}{y}+2021 \geq 11(x+y)+\frac{4}{x+y}+2021 $

$=(x+y)+\frac{4}{x+y}+10(x+y)+2021 $

$\geq 2 \sqrt{(x+y) \cdot \frac{4}{(x+y)}}+10.2+2021=2045$

b) Áp dụng bất đẳng thức Cauchy ta có:

$\frac{1}{x^{3}(2 y-x)}+x^{2}+y^{2}=\frac{1}{x^{2}\left(2 x y-x^{2}\right)}+x^{2}+y^{2} \geq \frac{1}{x^{2}\left(2 x y-x^{2}\right)}+2 x y $

$=\frac{1}{x^{2}\left(2 x y-x^{2}\right)}+x^{2}+\left(2 x y-x^{2}\right) \geq 3 \sqrt[3]{\frac{1}{x^{2}\left(2 x y-x^{2}\right)} \cdot x^{2} \cdot\left(2 x y-x^{2}\right)}=3$

Bài 3. Do phương trình $f(x)=x \Leftrightarrow a x^{2}+b x+c=x \Leftrightarrow a x^{2}+(b-1) x+c=0,(a \neq 0)$ vô nghiệm nên ta có:

$\Delta=(b-1)^{2}-4 a c<0 \Leftrightarrow(b-1)^{2}<4 a c$

Giả sử phương trình: $f(f(x))=x$ có nghiệm, gọi nghiệm đó là $x_{0}$, ta có:

$f\left(f\left(x_{0}\right)\right)=x_{0} \Leftrightarrow f\left(f\left(x_{0}\right)\right)-f\left(x_{0}\right)+\left[f\left(x_{0}\right)-x_{0}\right]=0 $

$\Leftrightarrow a\left[f\left(x_{0}\right)\right]^{2}+b f\left(x_{0}\right)-a x_{0}^{2}-b x_{0}+\left[f\left(x_{0}\right)-x_{0}\right]=0 $

$\Leftrightarrow a\left[f\left(x_{0}\right)-x_{0}\right]\left[f\left(x_{0}\right)+x_{0}\right]+b\left[f\left(x_{0}\right)-x_{0}\right]+\left[f\left(x_{0}\right)-x_{0}\right]=0 $

$\Leftrightarrow\left[f\left(x_{0}\right)-x_{0}\right]\left[a\left(f\left(x_{0}\right)+x_{0}\right)+b+1\right]=0 $

$\Leftrightarrow a\left(f\left(x_{0}\right)+x_{0}\right)+b+1=0 $

$\Leftrightarrow a^{2} x_{0}^{2}+a(b+1) x_{0}+a c+b+1=0$

Do đó phương trình: $a^{2} x^{2}+a(b+1) x+a c+b+1=0$ có nghiệm nên ta có:

$\Delta=a^{2}(b+1)^{2}-4 a^{2}(a c+b+1) \geq 0$

Từ đó dẫn đến

$(b+1)^{2}-4(a c+b+1) \geq 0 \Leftrightarrow 4 a c \leq b^{2}-2 b-3$

Suy ra: $b^{2}-2 b-3>(b-1)^{2} \Leftrightarrow b^{2}-2 b-3>b^{2}-2 b+1 \Leftrightarrow-4>0$ (vô lí). Do đó ta có điều phải chứng minh.

Bài 4. a) Lần lượt xét $x=0,1,2,3$ đều không nhận được $x=1,2,3$ là nghiệm. Do đó ta xét $x \geq 4$ và $x, y$ là hai số nguyên dương.

Vế trái chia hết cho 9 nên vế phải chia hết cho 9 , đặt: $y=3 z,\left(z \in N^{*}\right)$, ta có phương trình: $3^{x-2}+19=z^{2}$.

Nhận xét: $3 \equiv-1(\bmod 4)$ nên $3^{n} \equiv 1(\bmod 4)$, nếu $n$ chẵn và $3^{n} \equiv-1(\bmod 4)$, nếu $n$ lẻ.

Giả sử: Nếu $x$ là số lẻ thì $3^{x-2}+19 \equiv 18 \equiv 2(\bmod 4)$. Do một số chính phương chia 4 chỉ dư 0 hoặc 1 (vô lí).

b) Do đó khi $x$ là số chẵn thì $3^{x-2}+19 \equiv 20 \equiv 0(\bmod 4)$, suy ra $z$ là số chẳn. Đặt: $x-2=2 k,\left(k \in N^{*}\right)$. Ta có phương trình:

$3^{2 k}+19=z^{2} \Leftrightarrow z^{2}-3^{2 k}=19 \Leftrightarrow\left(z-3^{k}\right)\left(z+3^{k}\right)=19 $

$\Leftrightarrow\left\{\begin{array}{l}z+3^{k}=19 \\ z-3^{k}=1\end{array} \Leftrightarrow\left\{\begin{array}{c}z=10 \\ 3^{k}=9\end{array} \Leftrightarrow\left\{\begin{array}{l}z=10 \\ k=2\end{array} \Leftrightarrow\left\{\begin{array}{l}x=6 \\ y=30\end{array}\right.\right.\right.\right.$

Thử lại với $x=6, y=30$ (nhận). Do đó nghiệm duy nhất của phương trình là $(x ; y)=(6 ; 30)$.

Bài 5. a) Ta có: $\angle D P H=\angle E P H$ (tính chất hai tiếp tuyến cắt nhau) nên $\angle D G H=\angle E G H$, do đó hai cung $H D$ và cung $H E$ bằng nhau. Từ đó:

$\angle H C F=\angle H G E+\angle D E G=\angle H P D+\angle D P G=\angle H P G $

Dẫn đến, tứ giác $C F P G$ nội tiếp.

b) Ta có: $\angle O D I+\angle O E I=90^{\circ}+90^{\circ}=180^{\circ}$ nên tứ giác $D O E I$ nội tiếp.

c) Xét đường tròn $(P D E)$, với $H$ là điểm chính giữa cung $D E$ và $I$ là tâm đường tròn nội tiếp tam giác $\triangle P D E$, tính chất quen thuộc $H D=H I=H E$, do đó ta có $H$ là tâm đường tròn ngoại tiếp tứ giác $D O E I$.

Từ đó, $H$ là tâm đường tròn ngoại tiếp tam giác $\triangle D O E$.

d) Từ câu c) ta có $H O=H D=H I-H E$, lại có $\triangle H D C \sim \triangle H G D(\mathrm{~g}-\mathrm{g})$ nên $H D^{2}=H C . H G$, do đó $H O^{2}=H C . H G$. Suy ra $\triangle H O C \backsim \triangle H G O(\mathrm{c}-\mathrm{g}-\mathrm{c})$ nên $\angle H G O=\angle H O C$.

Lại có, $\angle H G P=\angle H F C$ nên $\angle O G P=\angle H G O+\angle H G P=\angle H O C+\angle H F C=90^{\circ}$, suy ra $A, G, P, B, O$ cùng thuộc một đường tròn.

Bài 6. a) Gọi $A$ là một điểm đã cho, nối $A$ với 16 điểm còn lại được 16 đoạn thẳng và chúng được tô bởi ba màu, Theo nguyên lý Dirichlet tồn tại ít nhất 6 đoạn thẳng có cùng một màu. Giả sử đó là các đoạn thẳng $A B, A C, A D, A E, A F, A G$ có cùng màu đỏ. Xét các đoạn thẳng nối từng cặp điểm trong 6 điểm $B, C, D, E, F, G$. Xảy ra các trường hợp sau:

– Trường hợp 1. Tồn tại một đoạn thẳng có màu đỏ, chẳng hạn $B C$, thì tam giác $\triangle A B C$ có ba cạnh cùng là màu đỏ, khẳng định đúng.

– Trường hợp 2. Tất cả các đoạn thẳng nối $B, C, D, E, F, G$ chỉ có màu xanh hoặc vàng. Ta xét 5 đoạn $B C, B D, B E, B F, B G$ được tô bởi hai màu thì theo nguyên lý Dirichlet tồn tại ít nhất 3 đoạn thẳng có cùng một màu. Giả sử là $B C, B D, B E$ cùng có màu xanh.

  • Nếu trong ba đoạn thẳng $C D, C E, D E$ có một đoạn tô màu xanh, chẳng hạn là $C D$ thì tam giác $\triangle B C D$ có ba cạnh cùng màu xanh, khẳng định đúng.

  • Nếu trong ba đoạn thẳng $C D, C E, D E$ không có một đoạn nào màu xanh, thì tam giác $\triangle C D E$ có ba cạnh cùng màu vàng, khẳng định đúng.

Vậy tồn tại tam giác có ba cạnh cùng một màu.

b) Chia mỗi số nguyên dương ghi trên các đoạn thẳng cho 3 ta được các số dư là $0,1,2$. Ta tô màu đoạn thẳng ghi số dư $0,1,2$ theo thứ tự úng với màu đỏ, xanh, vàng. Theo kết quả trên tồn tại một tam giác có ba cạnh cùng một màu, tức là ba số đó có cùng số dư $r$, chẳng hạn là $3 k+r, 3 h+r, 3 m+r$. Lúc đó tổng ba số trên ba cạnh của tam giác đó bằng:

$3 k+r+3 h+r+3 m+r=3(k+h+m+r) \vdots 3$

mà $3 k+r+3 h+r+3 m+r>3$ do đó $3 k+r+3 h+r+3 m+r$ là hợp số.

 

 

 

 

 

 

 

 

 

 

ĐỀ THI VÀO LỚP 10 KHÔNG CHUYÊN TRƯỜNG PHỔ THÔNG NĂNG KHIẾU 2021

Bài 1. (1,5 điểm) Cho biểu thức:

$P=\frac{a^{2}+b \sqrt{a b}}{a+\sqrt{a b}}+\frac{a \sqrt{a}-3 a \sqrt{b}+2 b \sqrt{a}}{\sqrt{a}-\sqrt{b}} \quad(a>b>0)$

a) Thu gọn biểu thức $P$.

b) Chứng minh $P>0$.

Bài 2. (2 điểm)

a) Giải phương trình: $\left(x^{2}+2 x-3\right)(\sqrt{3-2 x}-\sqrt{x+1})=0$

b) Cho $(d): y=(m+1) x+m n$ và $\left(d_{1}\right): y=3 x+1$. Tìm $m, n$ biết $(d)$ đi qua $A(0 ; 2)$, đồng thời $(d)$ song song với $\left(d_{1}\right)$.

Bài 3. (1,5 điểm) Cho $(P),(d)$ lần lượt là đồ thị hàm số $y=x^{2}$ và $y=2 x+m$.

a) Tìm $m$ sao cho $(P)$ cắt $(d)$ tại hai điểm phân biệt $A\left(x_{1} ; y_{1}\right), B\left(x_{2} ; y_{2}\right)$.

b) Tìm $m$ sao cho $\left(x_{1}-x_{2}\right)^{2}+\left(y_{1}-y_{2}\right)^{2}=5$.

Bài 4. (2 điểm)

a) Công ty viễn thông gói cước được tính như sau:

  • Gói I: 1800 đồng/phút cho 60 phút đầu tiên; 1500 đồng/phút cho 60 phút tiếp theo và 1000 đồng/phút cho thời gian còn lại.

  • Gói II: 2000 đồng/phút cho 30 phút đầu tiên; 1800 đồng/ phút cho 30 phút tiếp theo; 1200 đồng/phút cho 30 phút tiếp theo nữa và 800 đồng/phút cho thời gian còn lại.

Sau khi cân nhắc thì bác An chọn gói II vì sẽ tiết kiện được 95000 đồng so với gói I. Hỏi trung bình bác An gọi bao nhiêu phút một tháng?

b) Cho $\triangle A B C$ có $A B=3, A C=4, B C=5$. $B D$ là tia phân giác của $\angle A B C$. Tính $B D$ ?

Bài 5. (3 điểm) Cho $\triangle A B C$ nhọn $(A B<A C)$ nội tiếp đường tròn $(T)$ có tâm $O$, bán kính $R$, $B C=R \sqrt{3}$. Tiếp tuyến tại $B, C$ của $(T)$ cắt nhau tại $P$. Cát tuyến $P A$ cắt $(T)$ tại $D$ (khác $A$ ). Đường thẳng $O P$ cắt $B C$ tại $H$.

a) Chứng minh $\triangle P B C$ đều. Tính $P A \cdot P D$ theo $R$.

b) $A H$ cắt $(T)$ tại $E($ khác $A$ ). Chứng $\operatorname{minh} H A \cdot H E=H O \cdot H P$ và $P D=P E$.

c) Trên $A B$ lấy điểm $I$ thỏa $A I=A C$, trên $A C$ lấy điểm $J$ thỏa $A J=A B$. Đường thẳng vuông góc với $A B$ tại $I$ và đường thẳng vuông góc với $A C$ tại $J$ cắt nhau ở $K$. Chứng $\operatorname{minh} I J=B C$ và $A K \perp B C$. Tính $P K$ theo $R$.

LỜI GIẢI

Bài 1. a) Ta có $a>b>0$ nên

$P =\frac{a^{2}+b \sqrt{a b}}{a+\sqrt{a b}}+\frac{a \sqrt{a}-3 a \sqrt{b}+2 b \sqrt{a}}{\sqrt{a}-\sqrt{b}} $

$=\frac{(\sqrt{a})^{3}+(\sqrt{b})^{3}}{\sqrt{a}+\sqrt{b}}+\frac{(\sqrt{a}-\sqrt{b})(a-2 \sqrt{a b})}{\sqrt{a}-\sqrt{b}} $

$=a-\sqrt{a b}+b+a-2 \sqrt{a b} $

$=2 a-3 \sqrt{a b}+b .$

b) Ta có $a>b>0$ nên $\sqrt{a}>\sqrt{b}$, do đó

$P=2 a-3 \sqrt{a b}+b=(\sqrt{a}-\sqrt{b})(2 \sqrt{a}-\sqrt{b})>0 \text {. }$

Bài 2. a) $\left(x^{2}+2 x-3\right)(\sqrt{3-2 x}-\sqrt{x+1})=0 \quad(*)$

Điều kiện: $\left\{\begin{array}{l}3-2 x \geq 0 \\ x+1 \geq 0\end{array} \Leftrightarrow-1 \leq x \leq \frac{3}{2}\right.$

$(*) \Leftrightarrow(x-1)(x+3)(\sqrt{3-2 x}-\sqrt{x+1})=0$

$\Leftrightarrow\left[\begin{array}{c}x-1=0 \\ x+3=0 \ 3-2 x=x+1\end{array}\right.$

Vậy  $S=(1 ; \frac{2}{3})$

b) $(d) / /\left(d_{1}\right) \Leftrightarrow\left\{\begin{array}{l}m+1=3 \\ m \cdot n \neq 1\end{array} \Leftrightarrow\left\{\begin{array}{l}m=2 \\ n \neq \frac{1}{2}\end{array}\right.\right.$

$Vì  A(0 ; 2) \in(d): y=3 x+2 n \Leftrightarrow 2=3.0+2 n \Leftrightarrow n=1$

Vậy $m=2, n=1$

Bài 3. a) Phương trình hoành độ giao điểm của $(P)$ và $(d)$

$x^{2}=2 x+m \Leftrightarrow x^{2}-2 x-m=0$

$(P)$ cắt $(d)$ tại 2 diểm phân biệt $A, B \Leftrightarrow(1)$ có 2 nghiệm phân biệt

$\Leftrightarrow \Delta^{\prime}>0 \Leftrightarrow 1+m>0 $

$\Leftrightarrow m>-1(*)$

Vậy $m>-1$ thì $(P)$ cắt $(d)$ tại hai điểm phân biệt.

b) Với điều kiện $(*)$ theo Viet ta có: $S=x_{1}+x_{2}=2, P=x_{1} \cdot x_{2}=-m$ Ta có: $A\left(x_{1} ; y_{1}\right) \in(d) \Leftrightarrow y_{1}=2 x_{1}+m ; B\left(x_{2} ; y_{2}\right) \in(d) \Leftrightarrow y_{2}=2 x_{2}+m$ Ta có:

$\left(x_{1}-x_{2}\right)^{2}+\left(y_{1}-y_{2}\right)^{2}=5 $

$\Leftrightarrow\left(x_{1}-x_{2}\right)^{2}+\left(2 x_{1}-2 x_{2}\right)^{2}=5 $

$\Leftrightarrow\left(x_{1}-x_{2}\right)^{2}+4\left(x_{1}-x_{2}\right)^{2}=5 $

$\Leftrightarrow\left(x_{1}-x_{2}\right)^{2}=1 \Leftrightarrow\left(x_{1}+x_{2}\right)^{2}-4 x_{1} x_{2}=1 $

$\Leftrightarrow 4+4 m=1 \Leftrightarrow m=\frac{-3}{4}(\text { thỏa }(*)) $

Vậy  $m=-\frac{3}{4}$

Bài 4. a) Giả sử thời gian gọi trung bình mỗi tháng của bác An là $t($ phút, $t>0)$. Gọi $A(x), B(x)$ lần lượt là cước phí khi gọi $x$ phút tương ứng với gói cước I và gói cước II, theo đề bài ta có $A(t)-B(t)=95000$ (đồng).

Ta có bảng sau:

Vậy trung bình mỗi tháng bác An gọi 475 phút.

b) Ta có: $3^{2}+4^{2}=5^{2}$ nên $A B^{2}+A C^{2}=B C^{2}$

Theo định lý Pythagore đảo, tam giác $A B C$ vuông tại $A$.

Theo tính chất đường phân giác: $\frac{D C}{B C}=\frac{D A}{B A}$.

Suy ra $\frac{D C}{B C}=\frac{D A}{B A}=\frac{D C+D A}{B C+B A}=\frac{A C}{B A+B C}=\frac{1}{2} \Rightarrow A D=\frac{1}{2} B A=\frac{3}{2}$.

Tam giác $A B D$ vuông tại $A$ nên: $B D^{2}=A D^{2}+A B^{2}=\frac{45}{4} \Rightarrow B D=\frac{3 \sqrt{5}}{2}$.

Bài 5.

a) – Ta có: $O B=O C, P B=P C$ suy ra $P O$ là đường trung trực của $B C$ nên $O P \perp B C$ và $H$ là trung điểm $B C$.

$\sin \angle H O C=\frac{H C}{O C}=\frac{\sqrt{3}}{2} \Rightarrow \angle H O C=60^{\circ} \Rightarrow \angle H C P=\angle H O C=60^{\circ}$

$\triangle P B C$ có $P B=P C$ và $\angle B C P=60^{\circ}$ suy ra $\triangle P B C$ đều

  • Xét $\triangle P B D$ và $\triangle P A B$ có $\angle B P D$ chung, $\angle P B D=\angle P A B$

$\Rightarrow \triangle P B D \backsim \triangle P A B(\mathrm{~g} . \mathrm{g}) \Rightarrow \frac{P B}{P A}=\frac{P D}{P B} \Rightarrow P A \cdot P D=P B^{2}=3 R^{2}$

b)

  • Xét $\triangle H A B$ và $\triangle H C E$ có $\angle A H B=\angle C H E, \angle H A B=\angle H C E$

$\Rightarrow \triangle H A B \backsim \triangle H C E(g . g) \Rightarrow H A \cdot H E=H B \cdot H C=H B^{2}=H O \cdot H P$

  • Xét $\triangle H O A$ và $\triangle H E P$ có $\angle O H A=\angle E H P, \frac{H O}{H E}=\frac{H A}{H P}$ $\Rightarrow \triangle H O A \backsim \triangle H E P($ c.g.c $)$

$\Rightarrow \angle H O A=\angle H E P$, suy ra $A O E P$ là tứ giác nội tiếp.

Suy ra $\angle H P E=\angle H P D$ (chắn hai cung $O E$ và $O A$ bằng nhau)

Lại có $P A \cdot P D=P B^{2}=P H \cdot P O \Rightarrow \frac{P D}{P O}=\frac{P H}{P A}$ $\Rightarrow \triangle P D H \backsim \triangle P O A$ (c.g.c) suy ra $O H D A$ nội tiếp.

Mà $\angle P A O=\angle O D A=\angle A H O=\angle P H E$ nên $\angle P H D=\angle P H E$

Từ (1) và (2) suy ra $\triangle H D P=\triangle H E P$ (g.c.g), suy ra $P D=P E$.

c)

  • Xét $\triangle A B C$ và $\triangle A J I$ có $A B=A J, \angle I A C$ chung, $A C=A I$ nên $\triangle A B C=\triangle A J I \Rightarrow I J=B C$

  • Gọi $Q=B C \cap A K$

Ta có: $\angle A I K=\angle A J K=90^{\circ}$ nên $A I K J$ nội tiếp đường tròn đường kính $A K$ $\Rightarrow \angle A K I=\angle A J I$

Mà $\angle A J I=\angle A B C$ (do $\triangle A B C=\triangle A J I$ ) nên $\angle A K I=\angle A B C$.

Tứ giác $B Q K I$ có $\angle A K I=\angle A B C$ nên $B Q K I$ là tứ giác nội tiếp. $\Rightarrow \angle B I K+\angle B Q K=180^{\circ} \Rightarrow \angle B Q K=180^{\circ}-\angle B I K=180^{\circ}-90^{\circ}=90^{\circ}$

Suy ra $A K \perp B C$.

  • Vì $\triangle A B C=\triangle A I J$ nên bán kính đường tròn ngoại tiếp của hai tam giác này bằng nhau.

Mà $A K$ là đường kính của đường tròn ngoại tiếp $\triangle A I J$ nên $A K=2 R$.

$\triangle O C P$ vuông tại $C$ :

$\Rightarrow O P^{2}=O C^{2}+C P^{2}=R^{2}+(R \sqrt{3})^{2}=4 R^{2} $

$\Rightarrow O P=2 R \Rightarrow O P=A K .$

Ta có: $A K \perp B C, O P \perp B C$ nên $A K / / O P$.

Tứ giác $A O P K$ có $A K / / O P$ và $A K=O P$ nên $A O P K$ là hình bình hành, suy ra $P K=A O=R$.

Vậy $P K=R$.

 

 

 

 

 

 

 

 

 

 

 

 

 

 

ĐỀ THI VÀO LỚP 10 CHUYÊN TOÁN TRƯỜNG PHỔ THÔNG NĂNG KHIẾU 2021

Bài 1. Cho hệ phương trình: $\left\{\begin{array}{l}\sqrt{x-2}+\sqrt{y-1}=2 \\ x+y=m\end{array}\right.$

a) Giải hệ với $m=7$

b) Tìm $m$ sao cho hệ có nghiệm $(x, y)$

Bài 2. Cho $M=\frac{1}{a}+\frac{1}{b}+\frac{1}{c}, N=\frac{1}{b+c}+\frac{1}{c+a}+\frac{1}{a+b}, K=\frac{a}{b+c}+\frac{b}{c+a}+\frac{c}{a+b}$

a) Chứng minh nếu $M K=\frac{a^{2}+b^{2}+c^{2}}{a b c}$ thì $N=0$

b) Cho $M=K=4, N=1$. Tính tích $a b c$.

Bài 3. Cho dãy $n$ số thực $x_{1} ; x_{2} ; \ldots ; x_{n}(n \geq 5)$ thỏa: $x_{1} \leq x_{2} \leq \ldots \leq x_{n}$ và $x_{1}+x_{2}+\ldots x_{n}=1$

a) Chứng minh nếu $x_{n} \geq \frac{1}{3}$ thì $x_{1}+x_{2} \leq x_{n}$

b) Chứng minh nếu $x_{n} \leq \frac{2}{3}$ thì tìm được số nguyên dương $k<n$ sao cho

$\frac{1}{3} \leq x_{1}+x_{2}+\ldots+x_{k} \leq \frac{2}{3}$

Bài 4. a) Tìm tất cả các số tự nhiên $n$ sao cho $(2 n+1)^{3}+1$ chia hết cho $2^{2021}$

b) Tìm tất cả số tự nhiên $n$ và số nguyên tố $p$ sao cho $\frac{2 n+2}{p}$ và $\frac{4 n^{2}+2 n+1}{p}$ là các số nguyên. Chứng minh với $n$ và $p$ tìm được, các số nguyên trên không thể đồng thời là số chính phương.

Bài 5. Cho tam giác $A B C$ vuông tại $A$. Các điểm $E, F$ lần lượt thay đổi trên các cạnh $A B, A C$ sao cho $E F | B C$. Gọi $D$ là giao điểm của $B F$ và $C E, H$ là hình chiếu của $D$ lên $E F$. Đường tròn $(I)$ đường kính $E F$ cắt $B F, C E$ tại $M, N$. ( $M$ khác $F, N$ khác $E$ )

a) Chứng minh $A D$ và đường tròn ngoại tiếp $\triangle H M N$ cùng đi qua tâm $I$ của đường tròn tâm $I$.

b) Gọi $K, L$ lần lượt là hình chiếu vuông góc của $E, F$ lên $B C$ và $P, Q$ tương ứng là giao điểm của $E M, F N$ với $B C$. Chứng minh tứ giác $A E P L, A F Q K$ nội tiếp và $\frac{B P \cdot B L}{C Q \cdot C K}$ không đổi khi $E, F$ thay đổi.

c) Chứng minh nếu $E L$ và $F K$ cắt nhau trên đường tròn $(I)$ thì $E M$ và $F N$ cắt nhau trên đường thẳng $B C$.

Bài 6. Cho $N$ tập hợp $(N \geq 6)$, mỗi tập hợp gồm 5 chữ cái khác nhau được lấy từ 26 chữ cái $a$, $b, c, \ldots, x, y, z$.

a) Biết rằng trong $N$ tập hợp đã cho, hai tập hợp bất kỳ có chung đúng 1 chữ cái, và không có chữ cái nào có mặt trong tất cả $N$ tập hợp này.

Chứng minh không có chữ cái nào có mặt trong 6 tập hợp từ $N$ tập đã cho.

b) Biết rằng trong $N$ tập hợp đã cho, hai tập hợp bất kỳ có chung đúng 2 chữ cái, và không có hai chữ cái nào cùng xuất hiện trong $N$ tập hợp này.

Hỏi trong số $N$ tập hợp đã cho, có nhiều nhất bao nhiêu tập hợp có chung đúng 2 chữ cái?

 

LỜI GIẢI

 

Bài 1.

a) $\left\{\begin{array}{l}\sqrt{x-2}+\sqrt{y-1}=2 \\ x+y=m\end{array}\right.$ (1)

ĐKXĐ: $x \geq 2, y \geq 1$

(1) $\Leftrightarrow\left\{\begin{array}{l}x-2+y-1+2 \sqrt{(x-2)(y-1)}=4 \\ x+y=7\end{array}\right.$

$\Leftrightarrow\left\{\begin{array}{l}2 \sqrt{(x-2)(y-1)}=0 \\ x+y=7\end{array}\right.$

$\Leftrightarrow\left[\left\{\begin{array}{l}x-2=0 \\ x+y=7 \\ y-1=0 \\ x+y=7\end{array} \Leftrightarrow\left\{\left\{\begin{array}{l}x=2 \\ y=5 \\ y=1 \\ x=6\end{array}(n)\right.\right.\right.\right.$

Vậy $(x, y) \in[(2 ; 5),(6 ; 1)]$

b) Đặt $u=\sqrt{x-2}, v=\sqrt{y-1}(u, v \geq 0$

Hệ phương trình trở thành: $\left\{\begin{array}{l}u+v=2 \\ u^{2}+v^{2}=m-3\end{array}\right.$

$\Rightarrow 2 u^{2}-4 u+7-m=0$ (2)

Để hệ (1) có nghiệm thì (2) phải có nghiệm không âm, nhỏ hơn hoặc bằng 2 , khi và chỉ khi:

$\left\{\begin{array} { l }{ \Delta ^ { \prime } \geq 0 } \\ { S > 0 } \\ { P \geq 0 } \\ { ( x _ { 1 } – 2 ) ( x _ { 2 } – 2 ) > 0 } \\ { S \leq 4 }\end{array} \Leftrightarrow \left\{\begin{array}{l}m \geq 7 \\ m \leq 7\end{array}\right.\right.$

Vậy $5 \leq m \leq 7$ thì hệ đã cho có nghiệm $(x, y)$

Bài 2.

a) $M K=\frac{a^{2}+b^{2}+c^{2}}{a b c} \Rightarrow N=0$.

$M K =\left(\frac{1}{a}+\frac{1}{b}+\frac{1}{c}\right)\left(\frac{a}{b+c}+\frac{b}{c+a}+\frac{c}{a+b}\right) $

$=\frac{1}{b+c}+\frac{b}{a(c+a)}+\frac{c}{a(a+b)}+\frac{a}{b(b+c)}+\frac{1}{c+a}+\frac{c}{b(a+b)}+$

$ \frac{a}{c(b+c)}+\frac{b}{c(c+a)}+\frac{1}{a+b} $

$=N+\frac{b}{c+a}\left(\frac{1}{a}+\frac{1}{c}\right)+\frac{c}{a+b}\left(\frac{1}{a}+\frac{1}{b}\right)+\frac{a}{b+c}\left(\frac{1}{b}+\frac{1}{c}\right) $

$=N+\frac{b}{a c}+\frac{c}{a b}+\frac{a}{b c} $

$=N+\frac{a^{2}+b^{2}+c^{2}}{a b c}$

Mà $M K=\frac{a^{2}+b^{2}+c^{2}}{a b c} \Rightarrow N+\frac{a^{2}+b^{2}+c^{2}}{a b c}=\frac{a^{2}+b^{2}+c^{2}}{a b c} \Rightarrow N=0$

b) Ta có $M=K=4 ; N=1$

Theo câu a) ta được:

$M K=N+\frac{a^{2}+b^{2}+c^{2}}{a b c} \Rightarrow 16=1+\frac{a^{2}+b^{2}+c^{2}}{a b c} \Rightarrow a^{2}+b^{2}+c^{2}=15 a b c $

$\Rightarrow(a+b+c)^{2}-2(a b+b c+c a)=15 a b c(*)$

Ta có:

$K+3=\frac{a}{b+c}+1+\frac{b}{c+a}+1+\frac{c}{a+b}+1=(a+b+c) N \Rightarrow 7=a+b+c $

$M=4 \Rightarrow a b+b c+c a=4 a b c .$

Thay vào $(*) \Rightarrow 7^{2}-2.4 a b c=15 a b c \Rightarrow a b c=\frac{49}{23}$.

Bài 3.

a) Giả sử rằng $x_{1}+x_{2}>x_{n} \geq \frac{1}{3}>0$

$\Rightarrow x_{2}>0 \Rightarrow x_{i}>0, \forall i \geq 2 \text {. }$

Suy ra $x_{1}+x_{2}+x_{n-2}+x_{n-1}+x_{n} \leq x_{1}+x_{2}+\ldots+x_{n-2}+x_{n-1}+x_{n}=1$

Nhưng $x_{1}+x_{2}>\frac{1}{3}$ và $x_{n-1}, x_{n-2}>\frac{1}{2}\left(x_{1}+x_{2}\right)>\frac{1}{6}$ và $x_{n} \geq \frac{1}{3}$ nên khi cộng theo vế, ta có $V T>1$, vô lý.

Vậy điều giả sử là sai hay nếu $x_{n} \geq \frac{1}{3}$ thì $x_{1}+x_{2} \leq x_{n}$

b) Giả sử không tồn tại số $k$ như trên.

Khi đó tồn tại chỉ số $l \leq n-1$ để

$x_{1}+\ldots+x_{l}<\frac{1}{3} \text { và } x_{1}+\ldots+x_{l+1}>\frac{2}{3}$

Suy ra $x_{l+1}>\frac{1}{3} \Rightarrow x_{k}>\frac{1}{3}>0, \forall k \geq l+1$.

Nếu $l<n-1$ thì tồn tại $x_{l+2}$ do $l+2 \leq n$. Ta có

$x_{l+2} \geq x_{l+1}>\frac{1}{3} \Rightarrow\left(x_{1}+x_{2}+\ldots+x_{l+1}\right)+x_{l+2}>1$, vô lý do $x_{1}+\ldots+x_{n}=1$.

Từ đó $l=n-1$. Để ý rằng $x_{n} \leq \frac{2}{3}$ nên $x_{1}+\ldots+x_{n-1}=1-x_{n} \geq 1-\frac{2}{3}=\frac{1}{3}$.

Kết hợp với $l=n-1$ nên $x_{1}+\ldots+x_{n-1}>\frac{2}{3} \Rightarrow x_{n}<\frac{1}{3}$, vô lý.

Vậy điều giả sử là sai hay phải tồn tại chỉ số $k<n$ để:

$\frac{1}{3} \leq x_{1}+x_{2}+\cdots+x_{k} \leq \frac{2}{3}$

Bài 4.

(a) $(2 n+1)^{3}+1 \vdots 2^{2021} $

$\Leftrightarrow(2 n+2)\left(4 n^{2}+2 n+1\right) \vdots 2^{2021} $

$\Leftrightarrow 2(n+1)\left(4 n^{2}+2 n+1\right) \vdots 2^{2021} $

$\Leftrightarrow(n+1)\left(4 n^{2}+2 n+1\right) \vdots 2^{2020} $

$\Leftrightarrow n+1 \vdots 2^{2020} \quad\left(\text { do } 4 n^{2}+2 n+1 \equiv 1(\bmod 2)\right) $

$\Leftrightarrow n=2^{2020} k-1\left(k \in \mathbb{Z}^{+}\right)$

b) Từ $p \mid 2 n+2$ và $p \mid 4 n^{2}+2 n+1$ thì $p$ phải là số lẻ, dẫn đến $p \mid n+1$.

Do $4 n+2+2 n+1=4(n-1)(n+1)+2(n+1)+3$ nên $p \mid 3$, từ đó $p=3$. Kết hợp với điều kiện $p \mid n+1$ thì $n=3 k-1$ với $k \in \mathbb{Z}^{+}$.

Ta chứng minh rằng $\frac{2 n+2}{3}$ và $\frac{4 n+2+2 n+1}{3}$ không cùng là số chính phương. Thật vậy, giả sử rằng ta có điều ngược lại, vì chúng đều là số nguyên dương nên:

$\frac{2 n+2}{3} \cdot \frac{4 n^{2}+2 n+1}{3}=s^{2}\left(s \in \mathbb{Z}^{+}\right)$

Viết lại thành $(2 n+1)^{3}=(3 s-1)(3 s+1)$. Do $s$ là số chẵn nên $(3 s-1,3 s+1)=1$, dẫn đến việc tồn tại các số nguyên $a, b$ để $a b=2 n+1,(a, b)=1$ và:

$\left\{\begin{array}{l}3 s-1=a^{3} \\ 3 s+1=b^{3}\end{array}\right.$

Từ đây $2=(b-a)\left(b^{2}+b a+a^{2}\right)$. Do $b>a$ nên $b-a \in{1,2}$. Xét từng trường hợp và giải ra cụ thể, ta được $(a, b)=(-1,1)$. Tuy nhiên điều này dẫn đến $s=0$, trái với việc $s>0$ từ điều đã giả sử.

Vậy giả sử ban đầu là sai hay hai số đã cho không thể cùng là số chính phương.

Bài 5.

a) a. Qua $D$ vế đường thẳng song song $B C$ cắt $A B, A C$ tại $X, Y$.

Ta có $\frac{D Y}{B C}=\frac{D F}{B F}=\frac{D E}{E C}=\frac{D X}{B C}$.

Suy ra $D X=D Y$. Suy ra $D$ là trung điểm của $X Y$.

Do đó $A D$ qua trung điểm $I$ của $E F$.

Ta có $D H F N, D H E M$ nội tiếp. Suy ra $\widehat{D H N}=\widehat{D F N}=\widehat{M A N}$ và $\widehat{D H M}=$ $\widehat{N E M}=\widehat{N A M}$.

Suy ra $\widehat{M H N}=2 \widehat{M A N}=\widehat{M I N}$.

Suy ra tứ giác $M I H N$ nội tiếp. Ta có điều cần chứng minh.

b) Ta có $\triangle B M P \backsim \triangle B L F$. Suy ra $B M \cdot B F=B P \cdot B L$. Mặt khác $\triangle B A F \backsim \triangle B E M$, suy ra $B E \cdot B A=B M \cdot B E$.

Do đó $B A \cdot B E=B P \cdot B L$.

Từ đó ta có tứ giác $A E P L$ nội tiếp.

Chứng minh tương tự thì tứ giác $A F Q K$ nội tiếp.

Và $\frac{B P \cdot B L}{C Q \cdot C K}=\frac{B E \cdot B A}{C F \cdot C A}=\frac{A B^{2}}{A C^{2}}$.

c) Giả sử $E L, F K$ cắt nhau tại $S$ thuộc $(I)$. Khi đó $\angle E S F=90^{\circ}$ và $E F L K$ là hình vuông. Vẽ $P U \perp A B, Q V \perp A C$.

Ta có $\frac{B P}{B C}=\frac{B U}{B A}=\frac{B K}{B L}$ và $\frac{C Q}{B C}=\frac{C V}{C A}=\frac{C L}{C K}$ Đặt $x=E F=K L$

Ta cần chứng minh $\frac{B K}{B L}+\frac{C L}{C K}=1$.

$\Leftrightarrow B K \cdot C K+B L \cdot C L=B L \cdot C K $

$\Leftrightarrow B K(C L+x)+(B K+x) C L=(B K+x)(C L+x) \Leftrightarrow x^{2}=B K \cdot C L .$

Đúng vì tam giác $B E K$ và $C F L$ đồng dạng.

 

Bài 6.

a) Giả sử có chữ cái $\sigma$ sao cho $\sigma$ có mặt trong 6 tập hợp từ $N$ tập đã cho, chẳng hạn 6 tập $A_{1}, A_{2}, \ldots, A_{6}$.

Vì hai tập hợp bất kỳ có chung đúng một chữ cái nên hai tập hợp bất kỳ trong 6 tập trên bao giờ cũng chỉ có chũ cái chung duy nhất là $\sigma$.

Do đó, tổng số chữ cái có mặt trong 6 tập trên là: $1+6(5-1)=25$.

$-$ Nếu $N=6$ thì vô lý do $\sigma$ không xuất hiện trong tất cả $N$ tập hợp. Do đó $N \geq 7$.

$-$ Với $N \geq 7$, lấy tập $A_{7}$, có 2 khả năng:

$-$ $A_{7}$ chứa $\sigma$ : Vì $A_{7}$ và những tập $A_{1}, A_{2}, \ldots, A_{6}$ có chung đúng một chũ̃ cái $\sigma$ nên $A_{7}$ còn chứa 4 phần tử không nằm trong bất kỳ tập nào thuộc $A_{1}, A_{2}$, …, $A_{6}$.

Suy ra tổng số chữ cái trong 7 tập trên là: $1+7(5-1)=29>26$ (vô lý)

$-$ $A_{7}$ không chứa $\sigma$.

Khi đó $A_{7}$ sẽ có chung đúng 1 phần tử với mỗi tập $A_{1}, A_{2}, \ldots, A_{6}$ và 6 phần tử này phải khác nhau. (vì 6 tập $A_{1}, A_{2}, \ldots, A_{6}$ đã có chung $\sigma$ )

Do đó $A_{7}$ có ít nhất 6 phần tử. (vô lý).

Vậy không có chữ cái nào nằm trong 6 tập hợp từ $N$ tập hợp đã cho.

b) Giả sử có nhiều nhất $k$ tập hợp có chung đúng 2 chữ cái, chẳng hạn $a$ và $b$.

Khi đó dễ thấy $k \geq N-1$ nên tồn tại một tập hợp khác chưa được kể tên trong $k$ tập hợp trên, đặt là tập hợp $X, X$ không chứa ${a, b}$.

  • Nếu $X$ không chứa cả $a$ lẫn $b$. $X$ giao mỗi tập trong $k$ tập kia ở 2 phần tử khác nhau nên $2 k \leq 5 \Rightarrow k \leq 2$

  • Nếu $X$ chỉ chứa $a$, không chứa $b$.

Khi đó 4 phần tử còn lại giao với $k$ tập kia ở các phần tử khác nhau, mà $\mathrm{X}$ có 5 phần tử nên $k \leq 4$.

Vậy có nhiều nhất 4 tập hợp có chung đúng 2 chữ cái.

Để chỉ ra một ví dụ về khả năng có 4 tập hợp, xét $N=6$. Để thuận tiện, thay các chữ cái bằng các con số từ 1 đến 26 . Khi đó chọn bộ $N$ tập hợp như sau:

$\left\{\begin{array}{l}A_{1}={1,2,3,4,5} \ A_{2}={1,2,6,7,8} \\ A_{3}={1,2,9,10,11} \\ A_{4}={1,2,12,13,14} \\ A_{5}={1,3,6,10,13} \\ A_{6}={2,3,6,9,12}\end{array}\right.$

Bộ 6 tập hợp này thỏa mãn tất cả các điều kiện của bài toán.

 

Lời giải được thực hiện bởi nhóm giáo viên Star Education: thầy Nguyễn Tăng Vũ, thầy Nguyễn Ngọc Duy, thầy Vương Trung Dũng, thầy Lê Phúc Lữ, thầy Nguyễn Tấn Phát, Nguyễn Tiến Hoàng, Nguyễn Công Thành, Trần Tín Nhiệm, Châu Cẩm Triều, Lê Quốc Anh.

 

 

 

 

 

 

 

 

 

 

 

 

 

 

 

 

 

 

 

 

ĐỀ THI VÀO LỚP 10 CHUYÊN TOÁN TRƯỜNG PHỔ THÔNG NĂNG KHIẾU 2020

Bài 1. (2 điểm) Cho các phương trình: $x^{2}+a x+3=0$ và $x^{2}+b x+5=0$ với $a, b$ là tham số.

(a) Chứng minh nếu $a b \geq 16$ thì trong hai phương trình trên có ít nhất một phương trình có nghiệm.

(b) Giả sử hai phương trình trên có nghiệm chung $x_{0}$. Tìm $a, b$ sao cho $|a|+|b|$ có giá trị nhỏ nhất.

Bài 2. (1,5 điểm) Cho phương trình: $3 x^{2}-y^{2}=23^{n}$ với $n$ là số tự nhiên.

(a) Chứng minh nếu $n$ chẵn thì phương trình đã cho không có nghiệm nguyên $(x, y)$.

(b) Chứng minh nếu $n$ lẻ thì phương trình đã cho có nghiệm nguyên $(x, y)$.

Bài 3. (3,5 điểm) Cho đường tròn $(O)$, dây cung $B C$ không chứa tâm $O$ và điểm $A$ thay đổi trên cung lớn $B C$. Lấy các điểm $E$ và $F$ thỏa mãn: $\angle A B E=\angle C A E=$ $\angle A C F=\angle B A F=90^{\circ}$.

(a) Chứng minh rằng $A E \cdot A C=A F \cdot A B$ và điểm $O$ là trung điểm $E F$.

(b) Hạ $A D$ vuông góc với $E F(D \in E F)$. Chứng minh các tam giác $D A B$ và $D C A$ đồng dạng và điểm $D$ thuộc một đường tròn cố định.

(c) Gọi $G$ là giao điểm của $A D$ với đường tròn $(O)(G \neq A)$. Chứng minh $A D$ đi qua một điểm cố định và $G B \cdot A C=G C \cdot A B$.

(d) Gọi $K$ là tâm đường tròn ngoại tiếp tam giác $A E F$. Chứng minh $A K$ đi qua một điểm cố định.

Bài 4. (1,5 điểm) Cho số tự nhiên $a=3^{13} \cdot 5^{7} \cdot 7^{20}$

(a) Gọi $A$ là tập hợp các số nguyên dương $k$ sao cho $k$ là ước của $a$ và $k$ chia hết cho 105. Hỏi tập $A$ có bao nhiêu phần tử?

(b) Giả sử $B$ là một tập con bất kỳ của $A$ có 9 phần tử. Chứng minh ta luôn có thể tìm được 2 phần tử của $B$ sao cho tích của chúng là số chính phương.

Bài 5. (1,5 điểm) Cho hệ phương trình với $k$ là tham số:

$\left\{\begin{array}{l}\frac{x}{\sqrt{y z}}+\sqrt{\frac{x}{y}}+\sqrt{\frac{x}{z}}=k \\ \frac{y}{\sqrt{z x}}+\sqrt{\frac{y}{z}}+\sqrt{\frac{y}{x}}=k \\ \frac{z}{\sqrt{x y}}+\sqrt{\frac{z}{x}}+\sqrt{\frac{z}{y}}=k\end{array}\right.$

(a) Giải hệ với $k=1$.

(b) Chứng minh hệ vô nghiệm với $k \geq 2$ và $k \neq 3$.

 

LỜI GIẢI

 

Bài 1. ( 2 điểm) Cho các phương trình: $x^{2}+a x+3=0$ và $x^{2}+b x+5=0$ với $a, b$ là tham số.

(a) Chứng minh nếu $a b \geq 16$ thì trong hai phương trình trên có ít nhất một phương trình có nghiệm.

(b) Giả sử hai phương trình trên có nghiệm chung $x_{0}$. Tìm $a, b$ sao cho $|a|+|b|$ có giá trị nhỏ nhất.

Lời giải.

(a) Xét phương trình: $x^{2}+a x+3=0 \quad(1)$, ta có: $\Delta_{1}=a^{2}-12$.

Xét phương trình: $x^{2}+b x+5=0 \quad(2)$, ta có: $\Delta_{2}=b^{2}-20$

Ta có: $\Delta_{1}+\Delta_{2}=a^{2}+b^{2}-32 \geq 2 a b-32 \geq 0$

Vậy trong hai số $\Delta_{1}$ và $\Delta_{2}$ có ít nhất một số không âm hay một trong hai phương trình đã cho có nghiệm.

(b) Có hai cách giải tham khảo sau:

Cách 1. Vì $x_{0}$ là nghiệm chung của phương trình (1) và (2) nên phương trình $2 x^{2}+(a+b) x+8=0$ có nghiệm.

Suy ra: $\Delta=(a+b)^{2}-64 \geq 0 \Leftrightarrow|a+b| \geq 8$

Ta có: $|a|+|b| \geq|a+b| \geq 8$. Dấu ” $=$ ” xảy ra khi và chỉ khi: $\left\{\begin{array}{l}a b \geq 0 \\|a+b|=8\end{array}\right.$

  • Nếu $a+b=8$ thì $x_{0}=-2$, suy ra: $\left\{\begin{array}{l}(-2)^{2}-2 a+3=0 \\ (-2)^{2}-2 b+5=0\end{array} \Leftrightarrow\right.$

$\left\{\begin{array}{l}a=\frac{7}{2} \\ b=\frac{9}{2}\end{array}\right.$

  • Nếu $a+b=-8$ thì $x_{0}=2$, suy ra: $\left\{\begin{array}{l}2^{2}+2 a+3=0 \\ 2^{2}+2 b+5=0\end{array} \Leftrightarrow\left\{\begin{array}{l}a=-\frac{7}{2} \\ b=-\frac{9}{2}\end{array}\right.\right.$

Cách 2. Dễ thấy $x_{0} \neq 0$.

  • $(1) \Leftrightarrow-a=\frac{x_{0}^{2}+3}{x_{0}} \Leftrightarrow|a|=\frac{x_{0}^{2}+3}{\left|x_{0}\right|}$

$(2) \Leftrightarrow-b=\frac{x_{0}^{2}+5}{x_{0}} \Leftrightarrow|b|=\frac{x_{0}^{2}+5}{\left|x_{0}\right|}$

  • Suy ra $|a|+|b|=2\left|x_{0}\right|+\frac{8}{\left|x_{0}\right|} \geq 2 \sqrt{2\left|x_{0}\right| \cdot \frac{8}{\left|x_{0}\right|}}=8$ Dấu ” $=$ “xảy ra khi và chỉ khi: $x_{0}^{2}=4 \Leftrightarrow\left[\begin{array}{l}x_{0}=2 \ x_{0}=-2\end{array}\right.$ Với $x_{0}=2$ hoặc $x_{0}=-2$, lần lượt giải được $a=\frac{7}{2} ; b=\frac{9}{2}$ hoặc $a=-\frac{7}{2} ; b=-\frac{9}{2}$

Vậy giá trị nhỏ nhất của $|a|+|b|$ là 8 khi $a=\frac{7}{2} ; b=\frac{9}{2}$ hoặc $a=-\frac{7}{2} ; b=$ $-\frac{9}{2}$

 

Bài 2. (1,5 điểm) Cho phương trình: $3 x^{2}-y^{2}=23^{n}$ với $n$ là số tự nhiên.

(a) Chứng minh nếu $n$ chẵn thì phương trình đã cho không có nghiệm nguyên $(x, y)$.

(b) Chứng minh nếu $n$ lẻ thì phương trình đã cho có nghiệm nguyên $(x, y)$.

Lời giải.

(a) Ta nhận thấy 1 số chính phương $m=a^{2}$ khi chia cho 3 thì có số dư lần lượt là 0 hoặc 1 .

Nên tổng 2 số chính phương nếu chia hết cho 3 thì mỗi số đều phải chia hết cho $3 .$

Quay lại bài toán, do $n$ chẵn nên $23^{n}$ và $y^{2}$ đều là các số chính phương mà $23^{n}+y^{2}=3 x^{2} \vdots 3 \Rightarrow 23^{n} \vdots 3$ (vô lý)

Vậy $n$ chẵn thì phương trình đã cho không có nghiệm nguyên.

(b) Do $n$ lẻ $\Rightarrow n=2 k+1\left(k \in \mathbb{N}^{*}\right)$

Xét $\left\{\begin{array}{l}x=3 \cdot 23^{k} \\ y=2 \cdot 23^{k}\end{array} \Rightarrow 3 x^{2}-y^{2}=23^{2 k+1}=23^{n}\right.$

Vậy phương trình có nghiệm nguyên

 

Bài 3. (3,5 điểm) Cho đường tròn $(O)$, dây cung $B C$ không chứa tâm $O$ và điểm $A$ thay đổi trên cung lớn $B C$. Lấy các điểm $E$ và $F$ thỏa mãn: $\angle A B E=$ $\angle C A E=\angle A C F=\angle B A F=90^{\circ}$.

(a) Chứng minh rằng $A E \cdot A C=A F \cdot A B$ và điểm $O$ là trung điểm $E F$.

(b) Hạ $A D$ vuông góc với $E F(D \in E F)$. Chứng minh các tam giác $D A B$ và $D C A$ đồng dạng và điểm $D$ thuộc một đường tròn cố định.

(c) Gọi $G$ là giao điểm của $A D$ với đường tròn $(O)(G \neq A)$. Chứng minh $A D$ đi qua một điểm cố định và $G B \cdot A C=G C \cdot A B$.

(d) Gọi $K$ là tâm đường tròn ngoại tiếp tam giác $A E F$. Chứng minh $A K$ đi qua một điểm cố định.

Lời giải.

(a) Ta có $\angle B A E+\angle E A F=90^{\circ}$ và $\angle C A F+\angle E A F=90^{\circ}$.

Suy ra $\angle B A E=\angle C A F . \triangle A B E \backsim \triangle A C F$, suy ra $A E \cdot A C=A B \cdot A F$

Gọi $I$ là giao điểm của $B E$ và $C F$. Khi đó $A I$ là đường kính của $O$.

Tứ giác $A E I F$ là hình bình hành, $O$ là trung điểm $A I$ nên là trung điểm $E F$.

(b) Các tứ giác $A D B E, A D F C$ nội tiếp.

Khi đó $\angle A D B=\angle A E B=\angle A F C=\angle A C D . \angle A B D=\angle A E C=\angle I F E=$ $\angle A F C=\angle A D C$. Suy ra $\triangle A D B \sim \triangle A C D A$. (g.g)

Ta có $\angle B D C=2 \angle A D B=2 \angle A E B=2 \angle E I F=\angle B O C$.

Suy ra tứ giác $B D O C$ nội tiếp. $D$ thuộc đường tròn ngoại tiếp tam giác BOC cố định.

(d) Gọi $M$ là trung điểm của $B C$. Ta chứng minh $A, M, K$ thẳng hàng.

Ta chứng minh được $\angle D A E=\angle K A F\left(\angle 90^{\circ}-\angle A E D\right)$.

Gọi $T$ là trung điểm $C G$. Ta có $\triangle A C D \sim \triangle B C G$ suy ra $\triangle A B C \sim \triangle D C G$.

Từ đó ta có $\triangle A C M \backsim \triangle D C T$.

Khi đó $\angle C A M=\angle C D T=\angle A C D=\angle B A D$.

Mà $\angle C A M=\angle C A F+\angle F A M$ và $\angle B A D=\angle B A E+\angle E A D$.

Suy ra $\angle F A M=\angle E A D=\angle F A K$. Vậy $A, M, K$ thẳng hàng. $A K$ qua trung điểm $M$ của $B C$ cố định.

 

Bài 4. (1,5 điểm) Cho số tự nhiên $a=3^{13} \cdot 5^{7} \cdot 7^{20}$

(a) Gọi $A$ là tập hợp các số nguyên dương $k$ sao cho $k$ là ước của $a$ và $k$ chia hết cho 105 . Hỏi tập $A$ có bao nhiêu phần tử?

(b) Giả sử $B$ là một tập con bất kỳ của $A$ có 9 phần tử. Chứng minh ta luôn có thể tìm được 2 phần tử của $B$ sao cho tích của chúng là số chính phương.

Lời giải.

(a) $k: 105 \Rightarrow k$ chia hết cho $3,5,7$

$\Rightarrow k=3^{n} \cdot 5^{m} \cdot 7^{p} \text { với } m, n, p \text { nguyên dương }$

$\Rightarrow \text { có } 13 \cdot 7 \cdot 20=1820 \text { cách. }$

(b) Cách 1: Giả sử $B$ là tập hợp 9 số nguyên dương $a_{i}, i=\overline{1,9}$ với $a_{i}=3^{n_{i}} \cdot 5^{m_{i}} \cdot 7^{p_{i}}$ trong đó $0 \leq n_{i} \leq 13 ; 0 \leq m_{i} \leq 7$ và $0 \leq p_{i} \leq 20$

Do $B$ có 9 phân tử. Xét nguyên lý Dirichlet với tập các số $n_{i}$ thì ta có ít nhất 5 số hạng $a_{i}$ sao cho các số mũ $n_{i}$ của 3 tương ứng cùng tính chẵn lẻ.

Xét tiếp nguyên lý Dirichlet 5 số này cho số mũ $m_{i}$ của 5 tương ứng thì ta có ít nhất 3 số mà số mũ $m_{i}$ cũng cùng tính chẵn lẻ.

Với 3 số còn lại này ta cũng xét nguyên lý Dirichlet cho số mũ $p_{i}$ của 7 thì ta sẽ có ít nhất 2 số cũng tính chẵn lẻ.

Do 2 số được chọn này có số mũ cùng tính chẵn lẻ với cả các số 3,5 và 7 nên tích chúng lại sẽ là số chính phương.

– Cách 2: Ta chia 9 số từ tập $B$ vào 8 tập con như sau:

$B_{1}$= ( số mũ của 3,5,7 đều chẵn )

$B_{2}$= ( số mũ 3,5,7 đều lẻ )

$B_{3}$= ( số mũ của 3 chẵn; 5,7 đều lẻ )

$B_{4}$= ( số mũ của 5 chẵn; 3,7 lẻ )

$B_{5}$= ( số mũ của 7 chẵn; 3,5 lẻ )

$B_{6}$= ( số mũ của 3,5 đều chẵn; 7 lẻ )

$B_{7}$= ( số mũ của 3,7 đều chẵn; 5 lẻ )

$B_{8}$= ( số mữ của 5,7 đều chẵn; 3 lẻ )

Do có 8 tập mà có 9 số nên theo nguyên lý Dirichlet thì có ít nhất 2 số thuộc cùng một tập $B_{i}$ nên tích của chúng sẽ là một số chính phương.

Bài 5. (1,5 điểm) Cho hệ phương trình với $k$ là tham số:

$\left\{\begin{array}{l}\frac{x}{\sqrt{y z}}+\sqrt{\frac{x}{y}}+\sqrt{\frac{x}{z}}=k \\ \frac{y}{\sqrt{z x}}+\sqrt{\frac{y}{z}}+\sqrt{\frac{y}{x}}=k \\ \frac{z}{\sqrt{x y}}+\sqrt{\frac{z}{x}}+\sqrt{\frac{z}{y}}=k \ \text { (a) Giải hệ với } k=1\end{array}\right.$

(b) Chứng minh hệ vô nghiệm với $k \geq 2$ và $k \neq 3$.

Lời giải.

– Cách 1: Điều kiện $x, y, z$ cùng dấu đôi một.

Ta xét hệ phương trình với $k \geq 1$

Hệ phương trình $\Leftrightarrow\left\{\begin{array}{l}x+\sqrt{x z}+\sqrt{x y}=k \sqrt{y z} \\ y+\sqrt{x y}+\sqrt{y z}=k \sqrt{z x} \\ z+\sqrt{z y}+\sqrt{z x}=k \sqrt{x y}\end{array}\right.$

Đặt $a=\sqrt{x y}, b=\sqrt{y z}, c=\sqrt{z x}(a, b, c>0)$

  • Trường hợp 1: $x, y, z>0 \Rightarrow x=\frac{a c}{b} ; y=\frac{a b}{c} ; z=\frac{b c}{a}$ Hệ phương trình $\Leftrightarrow\left\{\begin{array}{l}\frac{a c}{b}+a+c=k b \\ \frac{a b}{c}+a+b=k c \\ \frac{b c}{a}+b+c=k a\end{array} \Rightarrow\left\{\begin{array}{l}k a^{2}=a b+a c+b c(1) \\ k b^{2}=a b+b c+c a(2) \\ k c^{2}=a b+a c+b c(3)\end{array}\right.\right.$ Lấy (1)-(2): $k\left(a^{2}-b^{2}\right)=0 \Leftrightarrow a^{2}=b^{2} \Leftrightarrow\left\{\begin{array}{l}a=b \\ a=-b \text { (loại) }\end{array}\right.$

Tương tự lấy (2)-(3): $b=c$

Vậy $a=b=c \Rightarrow k a^{2}=3 a^{2} \Rightarrow k=3$

  • Trường hợp 2: $x, y, z<0 \Rightarrow x=-\frac{a c}{b} ; y=-\frac{a b}{c} ; z=-\frac{b c}{a}$

Hệ phương trình $\Rightarrow\left\{\begin{array}{l}k a^{2}=a b+a c-b c \\ k b^{2}=a b+b c-c a \\ k c^{2}=a c+b c-a b\end{array}\right.$

Cộng các phương trình lại ta có: $k\left(a^{2}+b^{2}+c^{2}\right)=a b+b c+a c$ mà $a b+b c+c a \leq a^{2}+b^{2}+c^{2}$

Suy ra $k\left(a^{2}+b^{2}+c^{2}\right) \leq a^{2}+b^{2}+c^{2} \Leftrightarrow k \leq 1$

Vậy $k=1$ và $a=b=c \Leftrightarrow x=y=z<0$

Câu a) Áp dụng điều trên, hệ có nghiệm $x=y=z<0$.

Câu b) Suy ra điều phải chứng minh.

– Cách 2: Điều kiện xác định là: $x, y, z$ cùng dương hoặc cùng âm.

Đặt $a=\sqrt{\frac{x}{y}}, b=\sqrt{\frac{y}{z}}, c=\sqrt{\frac{z}{x}}$ thì $a, b, c>0$ và $a b c=1$.

Ta có: $\frac{a}{c}=\frac{|x|}{\sqrt{y z}}, \frac{b}{a}=\frac{|y|}{\sqrt{z x}}, \frac{c}{b}=\frac{|z|}{\sqrt{x y}}$.

(a) Khi $k=1$, nếu $x, y, z>0$ thì $\frac{a}{c}+a+\frac{1}{c}=\frac{b}{a}+b+\frac{1}{a}=\frac{c}{b}+c+\frac{1}{b}=1$. Cộng lại suy ra $\left(a+\frac{1}{a}\right)+\left(b+\frac{1}{b}\right)+\left(c+\frac{1}{c}\right)+\left(\frac{a}{c}+\frac{b}{a}+\frac{c}{b}\right)=3$ Theo bất đẳng thức Cô-si thì rõ ràng $a+\frac{1}{a} \geq 2, b+\frac{1}{b} \geq 2, c+\frac{1}{c} \geq 2$ nên đẳng thức trên không thể xảy ra.

Xét trường hợp $x, y, z$ cùng âm thì $-\frac{a}{c}+a+\frac{1}{c}=-\frac{b}{a}+b+\frac{1}{a}=-\frac{c}{a}+c+\frac{1}{b}=1$

Trừ vào các vế và phân tích, ta suy ra: $\frac{(a-1)(b-1)}{a}=\frac{(b-1)(c-1)}{b}=\frac{(c-1)(a-1)}{c}=0$

Từ đây dễ dàng suy ra ít nhất 2 trong $a, b, c$ phải là 1 mà $a b c=1$ nên

$a=b=c=1$. Vì thế nên thay vào ta có $x=y=z<0$. Và mọi bộ số như thế đều thỏa mãn hệ.

(b) Với $k \geq 2$, giả sử hệ có nghiệm $(x, y, z)$. Nếu như $x, y, z<0$ thì ta có $\frac{(a-1)(b-1)}{a}=\frac{(b-1)(c-1)}{b}=\frac{(c-1)(a-1)}{c}=k-1>0 .$

Từ đó suy ra $a-1, b-1, c-1$ đều cùng dấu, kéo theo $a, b, c>1$ hoặc $a, b, c<1$ Tuy nhiên $a b c=1$ nên điều này không thể xảy ra.

Do đó, ta phải có $a, b, c>0$ nên đưa về

$\frac{a}{c}+a+\frac{1}{c}=\frac{b}{a}+b+\frac{1}{a}=\frac{c}{b}+c+\frac{1}{b}=k$

Trong các số $a, b, c$ giả sử $a=\max {a, b, c}$ thì $k=\frac{a}{c}+a+\frac{1}{c} \geq$ $\frac{a}{c}+2 \sqrt{\frac{a}{c}} \geq 1+2=3$ nên ta cần có $k \geq 3$. Vì $k \neq 3$ nên $k>3$.

Vì $a=\max {a, b, c} \geq 1$ nên ta có $2 b+1 \geq \frac{b}{a}+b+\frac{1}{a}=k>3$ kéo theo $b>1$. Tương tự từ $2 c+1>\frac{c}{b}+c+\frac{1}{b}=k>3$ nên $c>1$. Từ đây suy ra $a, b, c>1$ trong khi $a b c=1$, vô lý.

Vậy hệ luôn vô nghiệm với $k \geq 2$ và $k \neq 3$.

 

 

 

 

 

 

 

 

 

 

 

 

 

 

 

 

 

 

 

 

 

 

 

 

 

 

 

 

 

 

 

 

 

 

 

 

 

 

 

 

ĐỀ THI VÀO LỚP 10 CHUYÊN TOÁN PHỔ THÔNG NĂNG KHIẾU 2018

Bài 1. Cho các phương trình $x^{2}-x+m=0$

(1) và $m x^{2}-x+1=0$

(2) với $m$ là tham số.

(a) Tìm $m$ để các phương trình (1) và (2) đều có 2 nghiệm dương phân biệt.

(b) Giả sử điều kiện ở câu a) được thỏa mãn gọi $x_{1}$; $x_{2}$ là nghiệm của (1) và $x_{3} ; x_{4}$ là nghiệm của (2).

Chứng minh rằng $x_{1} x_{2} x_{3}+x_{2} x_{3} x_{4}+x_{3} x_{4} x_{1}+x_{4} x_{1} x_{2}>5$

Bài 2. Cho $a, b$ là hai số nguyên thỏa mãn $a^{3}+b^{3}>0$.

(a) Chứng minh rằng $a^{3}+b^{3} \geq a+b>0$.

(b) Chứng minh rằng $a^{3}+b^{3} \geq a^{2}+b^{2}$.

(c) Tìm tất cả các bộ số $x, y, z, t$ nguyên sao cho $x^{3}+y^{3}=z^{2}+t^{2}$ và $z^{3}+t^{3}=$ $x^{2}+y^{2}$.

Bài 3. Cho $A_{n}=2018^{n}+2032^{n}-1964^{n}-1984^{n}$ với $n$ là số tự nhiên.

(a) Chứng minh với mọi số tự nhiên $n$ thì $A_{n}$ chia hết cho 51 .

(b) Tìm tất cả những số tự nhiên $n$ sao cho $A_{n}$ chia hết cho 45 .

Bài 4. Cho tam giác $A B C$ nhọn. Một đường tròn qua $B, C$ cắt các cạnh $A B, A C$ lần lượt tại $E$ và $F ; B F$ cắt $C E$ tại $D$. Lây điểm $K$ sao cho từ giác $D B K C$ là hình bình hành.

(a) Chứng minh rằng $\triangle K B C$ đồng dạng với $\triangle D F E, \triangle A K C$ dồng dạng với $\triangle A D E$.

(b) Hạ $D M$ vuông góc với $A B, D N$ vuông góc với $A C$. Chứng minh rằng $M N$ vuông góc với $A K$.

(c) Gọi $I$ là trung điểm $A D$, $J$ là trung điểm $M N$. Chứng minh rằng đường thẳng $I J$ đi qua trung điểm của cạnh $B C$.

(d) Đường thẳng $I J$ cắt đường tròn ngoại tiếp tam giác $I M N$ tại $T(T \neq I)$. Chứng minh rằng $A D$ tiếp xúc với đường tròn ngoại tiếp tam giác $D T J$.

Bài 5. Đội văn nghệ của một trường THCS có 8 học sinh. Nhà trường muốn thành lập các nhóm tốp ca, mỗi nhóm gồm đúng 3 học sinh, (mỗi học sinh có thể tham gia vài nhóm tốp ca khác nhau). Biết rằng hai nhóm tốp ca bất kỳ có chung nhau nhiều nhất là một học sinh.

(a) Chứng minh rằng không có học sinh nào tham gia từ 4 nhóm tốp ca trở lên.

(b) Có thể thành lập nhiều nhất là bao nhiêu nhóm tốp ca như vậy?

LỜI GIẢI

 

Bài 1. Cho các phương trình $x^{2}-x+m=0 \quad$ (1) và $m x^{2}-x+1=0$

(2) với $m$ là tham số.

(a) Tìm $m$ để các phương trình (1) và $(2)$ đều có 2 nghiệm dương phân biệt.

(b) Giả sử điều kiện ở câu a) được thỏa mãn gọi $x_{1}$; $x_{2}$ là nghiệm của (1) và $x_{3} ; x_{4}$ là nghiệm của $(2)$.

Chứng minh rằng $x_{1} x_{2} x_{3}+x_{2} x_{3} x_{4}+x_{3} x_{4} x_{1}+x_{4} x_{1} x_{2}>5$

Lời giải.

(a) Xét phương trình (1): $x^{2}-x+m=0$

Phương trình (1) có hai nghiệm dương phân biệt:

$\left\{\begin{array}{l}\Delta>0 \\ S>0 \ P>0\end{array} \Leftrightarrow\left\{\begin{array}{l}1-4 m>0 \\ 1>0 \ m>0\end{array} \Leftrightarrow\left\{\begin{array}{l}m<\frac{1}{4} \\m>0\end{array} \Leftrightarrow 0<m<\frac{1}{4}\right.\right.\right.$

Phương trình (2) có hai nghiệm dương phân biệt:

$\left\{\begin{array}{l}m \neq 0 \\ \Delta>0 \\ S>0 \\ P>0\end{array} \Leftrightarrow\left\{\begin{array}{l}m \neq 0 \\ 1-4 m>0 \\ \frac{1}{m}>0 \\ \frac{1}{m}>0\end{array} \Leftrightarrow\left\{\begin{array}{l}m \neq 0 \\ m<\frac{1}{4} \\ m>0\end{array} \Leftrightarrow 0<m<\frac{1}{4}\right.\right.\right.$

Vậy để $(1)$ và $(2)$ có hai nghiệm dương phân biệt thì $0<m<\frac{1}{4}$

b) Theo Viet ta có: $\left\{\begin{array}{l}x_{1}+x_{2}=1 \\ x_{1} x_{2}=m \\ x_{3}+x_{4}=\frac{1}{m} \\ x_{3} x_{4}=\frac{1}{m}\end{array}\right.$

$\text { Ta có } x_{1} x_{2} x_{3}+x_{2} x_{3} x_{4}+x_{3} x_{4} x_{1}+x_{4} x_{1} x_{2}$

$=x_{1} x_{3}+\frac{x_{2}}{m}+\frac{x_{1}}{m}+m x_{4}$

$=m\left(x_{3}+x_{4}\right)+\frac{1}{m}\left(x_{1}+x_{2}\right)$

$=1+\frac{1}{m}>1+\frac{1}{\frac{1}{4}}=5(\text { dpcm }) .$

Bài 2. Cho $a, b$ là hai số nguyên thỏa mãn $a^{3}+b^{3}>0$.

(a) Chứng minh rằng $a^{3}+b^{3} \geq a+b>0$.

(b) Chứng minh rằng $a^{3}+b^{3} \geq a^{2}+b^{2}$.

(c) Tìm tất cả các bộ số $x, y, z, t$ nguyên sao cho $x^{3}+y^{3}=z^{2}+t^{2}$ và $z^{3}+t^{3}=x^{2}+y^{2} .$

Lời giải. $a, b \in \mathbb{Z}: a^{3}+b^{3}>0$

(a) $a^{3}+b^{3}>0 \Leftrightarrow(a+b)\left(a^{2}-a b+b^{2}\right)>0$

Do $a^{2}-a b+b^{2}=\left(a-\frac{b}{2}\right)^{2}+\frac{3 b^{2}}{4} \geq 0$. Dấu “=” xảy ra $\Leftrightarrow a=b=0$ (loại).

$\Rightarrow a^{2}-a b+b^{2}>0$ nên $a+b>0$ (đpcm).

Ta có: $a^{3}+b^{3} \geq a+b$

$\Leftrightarrow(a+b)\left(a^{2}-a b+b^{2}-1\right) \geq 0 \quad (* *)$

Do $\left\{\begin{array}{l}a^{2}-a b+b^{2}>0 \\ a, b \in \mathbb{Z}\end{array} \Rightarrow a^{2}-a b+b^{2} \geq 1\right.$ nên $(* *)$ đúng.

Vậy $a^{3}+b^{3} \geq a+b$ và dấu “=” xảy ra $\Leftrightarrow\left\{\begin{array}{l}a=1 \\ b=0\end{array}\right.$ hay $\left\{\begin{array}{l}a=0 \\ b=1\end{array}\right.$ hay $\left\{\begin{array}{l}a=1 \\ b=1\end{array}\right.$

(b) Cách 1:

Do $a+b>0 \Rightarrow a+b \geq 1$.

TH1: $a+b=1 \Leftrightarrow b=1-a$.

Ta có: $a^{3}+b^{3} \geq a^{2}+b^{2} \Leftrightarrow a^{3}+(1-a)^{3} \geq a^{2}+(1-a)^{2}$

$\Leftrightarrow a^{2}-a \geq 0$

$\Leftrightarrow a \leq 0$ hoặc $a \geq 1$ (đúng vì $a \in \mathbb{Z}$ )

Vậy $a^{3}+b^{3} \geq a^{2}+b^{2}$ và dấu “=” xảy ra $\Leftrightarrow(a ; b) \in{(0 ; 0) ;(1 ; 1) ;(0 ; 1) ;(1 ; 0)}$.

TH2: $a+b \geq 2$

Ta có: $a^{3}+b^{3}=(a+b)\left(a^{2}-a b+b^{2}\right) \geq 2\left(a^{2}-a b+b^{2}\right)=a^{2}+b^{2}+$ $(a-b)^{2} \geq a^{2}+b^{2}$.

Cách 2:

Rõ ràng $a, b$ không thể đồng thời bé hơn 0 .

TH1: $a=b=0$ : hiển nhiên $a^{3}+b^{3} \geq a^{2}+b^{2}$

TH2: Một trong hai số bằng 0 , số còn lại khác 0 .

Giả sử: $\left\{\begin{array}{l}b=0 \\ a \neq 0\end{array} \Rightarrow a>1 \Rightarrow a^{3} \geq a^{2} \Rightarrow a^{3}+b^{3} \geq a^{2}+b^{2}\right.$

Dấu “=” xảy ra khi $a=1$.

TH3: $a, b \geq 1 \Rightarrow\left\{\begin{array}{l}a^{3} \geq a^{2} \\ b^{3} \geq b^{2}\end{array} \Rightarrow a^{3}+b^{3} \geq a^{2}+b^{2}\right.$

TH4: $\left\{\begin{array}{l}a>0 \\ b<0\end{array} \Rightarrow\left\{\begin{array}{l}a \geq 1 \\ b \leq-1\end{array}\right.\right.$

Đặt $a=|b|+k, k>1$

$a^{3}+b^{3} \geq a^{2}+b^{2}$

$\Leftrightarrow(|b|+k)^{3}+b^{3} \geq(|b|+k)^{2}+b^{2}$

$\Leftrightarrow 3|b|^{2} k+3|b| k^{2}+k^{3} \geq 2|b|^{2}+2|b| k+k^{2}$

$\left.\Rightarrow 3 b^{2} k+3|b| k+k^{3} \geq 2 b^{2}+2|b| k+k^{2} \quad \text { (Do k }>1\right)$

$\Leftrightarrow(3 k-2) b^{2}+|b| k+k^{2}(k-1) \geq 0 \text { (đúng). }$

Vậy $a^{3}+b^{3} \geq a^{2}+b^{2}$.

(c) Từ giả thiết $\Rightarrow x^{3}+y^{3} \geq 0 ; z^{3}+t^{3} \geq 0$.

Nếu $x^{3}+y^{3}=0 \Rightarrow z^{2}+t^{2}=0 \Rightarrow z=t=0$

$\Rightarrow x^{2}+y^{2}=0 \Rightarrow x=y=0 \text {. }$

Nếu $z^{3}+t^{3}=0$, tương tự ta có $x=y=z=t=0$.

Nếu $\left\{\begin{array}{l}x^{3}+y^{3}>0 \\ z^{3}+t^{3}>0\end{array}\right.$

Từ giả thiết suy ra $\left(x^{3}+y^{3}\right)+\left(z^{3}+t^{3}\right)=x^{2}+y^{2}+z^{2}+t^{2}(* * *)$

Theo câu b) : $\left\{\begin{array}{l}x^{3}+y^{3} \geq x^{2}+y^{2} \\ z^{3}+t^{3} \geq z^{t}+t^{2}\end{array}\right.$

Nếu $(* * *) \Leftrightarrow(x ; y),(z, t)$ là một trong các bộ $(1 ; 1) ;(1 ; 0) ;(0 ; 1)$.

Vậy nghiệm phương trình:

$(x, y, z, t) \in{(0 ; 0 ; 0 ; 0),(1 ; 1 ; 1 ; 1),(1 ; 0 ; 0 ; 1),(0 ; 1 ; 1 ; 0),(1 ; 0 ; 1 ; 0),(0 ; 1 ; 0 ; 1)} \text {. }$

Bài 3. Cho $A_{n}=2018^{n}+2032^{n}-1964^{n}-1984^{n}$ với $n$ là số tự nhiên.

(a) Chứng minh với mọi số tự nhiên $n$ thì $A_{n}$ chia hết cho 51 .

(b) Tìm tất cả những số tự nhiên $n$ sao cho $A_{n}$ chia hết cho 45 .

Lời giải.

(a) Do $2018 \equiv 1964 \quad(\bmod 3) \Rightarrow 2018^{n} \equiv 1964^{n} \quad(\bmod 3)$. $2032 \equiv 1984 \quad(\bmod 3) \Rightarrow 2032^{n} \equiv 1984^{n} \quad(\bmod 3) .$

$\Rightarrow A_{n} \vdots 3 .$

Ta lại có $2018 \equiv 1984 \quad(\bmod 17) \Rightarrow 2018^{n} \equiv 1984^{n} \quad(\bmod 17)$. $2032 \equiv 1964 \quad(\bmod 17) \Rightarrow 2032^{n} \equiv 1964^{n} \quad(\bmod 17) .$ $\Rightarrow A_{n} \vdots 17 .$

Do $(3 ; 17)=1$ nên $A_{n}: 51 \quad \forall n$

(b) $A_{n}=2018^{n}+2032^{n}-1964^{n}-1984^{n}$.

  • Ta xét các trường hợp của $n$ để $A_{n} \vdots 5$.

Ta có $A_{n} \equiv(-2)^{n}+2^{n}-2 \cdot(-1)^{n}(\bmod 5)$.

Do đó nếu $n$ lẻ $\Rightarrow A_{n} \equiv 2 \quad(\bmod 5) \quad$ (loại).

Nếu $n=4 k \Rightarrow A_{n} \equiv 2 \cdot 2^{4 k}-2 \equiv 2-2 \equiv 0 \quad(\bmod 5)$ (nhận)

Nếu $n=4 k+2 \Rightarrow A_{n} \equiv 2 \cdot 2^{4 k+2}-2 \equiv 8-2 \equiv 6(\bmod 5)$ (loại). Vậy $A_{n} \vdots 5 \Leftrightarrow n \vdots 4$.

  • Ta xét các trường hợp của $n$ để $A_{n}: 9$.

Ta có

$\begin{aligned} A_{n} & \equiv 2^{n}+(-2)^{n}-2^{n}-4^{n} \quad(\bmod 9) \\ & \equiv 2^{n}-4^{n} \quad(\bmod 9) \quad(\text { Do n chẵn }) \\ & \equiv 2^{n}\left(1-2^{n}\right) \quad(\bmod 9) \end{aligned}$

$\operatorname{Vi}(2 ; 9)=1 \Rightarrow 2^{n}-1: 9 .$

Xét $n=3 k$ với $k \in \mathbb{N}$. Ta có $A_{n} \equiv 2^{3 k}-1 \equiv(-1)^{k}-1 \quad(\bmod 9) \Rightarrow k$ chẵn

Xét $n=3 k+1$ với $k \in \mathbb{N}$. Ta có $A_{n} \equiv 2^{3 k+1}-1 \equiv 2 \cdot(-1)^{k}-$ $1(\bmod 9)$ (loại).

Xét $n=3 k+2$ với $k \in \mathbb{N}$. Ta có $A_{n} \equiv 2^{3 k+2}-1 \equiv 4 \cdot(-1)^{k}-$ $1(\bmod 9)$ (loại).

Vậy $A_{n} \vdots 45 \Leftrightarrow n \vdots 12$.

Bài 4. Cho tam giác $A B C$ nhọn. Một đường tròn qua $B, C$ cắt các cạnh $A B, A C$ lần lượt tại $E$ và $F ; B F$ cắt $C E$ tại $D$. Lấy điểm $K$ sao cho từ giác $D B K C$ là hình bình hành.

(a) Chứng minh rằng $\triangle K B C$ đồng dạng với $\triangle D F E, \triangle A K C$ đồng dạng với $\triangle A D E$.

(b) Hạ $D M$ vuông góc với $A B, D N$ vuông góc với $A C$. Chứng minh rằng

$M N$ vuông góc với $A K$.

(c) Gọi $I$ là trung điểm $A D, J$ là trung điểm $M N$. Chứng minh rằng đường thẳng $I J$ đi qua trung điểm của cạnh $B C$.

(d) Đường thẳng $I J$ cắt đường tròn ngoại tiếp tam giác $I M N$ tại $T(T \neq I)$. Chứng minh rằng $A D$ tiếp xúc với đường tròn ngoại tiếp tam giác $D T J$.

Lời giải.

(a) Tứ giác $B E F C$ nội tiếp nên $\angle D E F=\angle D B C$ và $\angle D F E=\angle D C B$.

Và $B D C K$ là hình bình hành nên $\angle D B C=\angle K C B, \angle D C B=\angle K B C$

Do đó $\angle D E F=\angle K C B, \angle D F E=\angle K B C$, suy ra $\triangle K B C \sim \triangle D F E$

Ta có $\angle A E C=\angle A B K$ và $\angle A B K=\angle A B D+\angle D B K=\angle A C E+\angle D C K=$ $\angle A C K$ (do $\angle A B D=\angle A C E, \angle D B K=\angle D C K)$

Do $\triangle D E F \sim \triangle K C B$ nên $\frac{D E}{C K}=\frac{E F}{B C}$ (1)

Mặt khác $\triangle A E F \sim \triangle A C B$ nên $\frac{E F}{B C}=\frac{A E}{A C}$ (2)

Từ (1) và (2) suy ra $\frac{D E}{C K}=\frac{A E}{A C}$

Xét $\triangle A E D$ và $\triangle A C K$ có $\angle A E D=\angle A C K, \frac{D E}{C K}=\frac{A E}{A C}$

Suy ra $\triangle A E D \sim \triangle A C K(\mathrm{c}-\mathrm{g}-\mathrm{c})$

(b) Gọi $Q$ là giao điểm của $A K$ và $M N$

Ta có $\triangle D A E \backsim \triangle K A C$ nên $\angle K A C=\angle D A E$ hay $\angle Q A C=\angle D A M$

Tứ giác $A M D N$ có $\angle A M D+\angle A N D=90^{\circ}+90^{\circ}=180^{\circ}$ nên nội tiếp.

Suy ra $\angle D N M=\angle D A M=\angle Q A N$

Mà $\angle D N M+\angle M N A=90^{\circ}$, suy ra $\angle Q A N+\angle M N A=90^{\circ}$

Suy ra $\angle A Q N=90^{\circ}$. Vậy $A K \perp M N$.

(c) Cách 1. Ta có $I J \perp M N$ và $A K \perp M N$, suy ra $I J | A K$.

Mà $I$ là trung điểm $A D$, suy ra $I J$ qua trung điểm $P$ của $D K$. Lại có $D B K C$ là hình bình hành nên $P$ cũng là trung điểm $B C$.

Cách 2. Gọi $P$ là trung điểm của $B C$. $V, U$ lần lượt là trung điểm của $D B, D C$.

Ta có $M I=\frac{1}{2} A D=N I$, suy ra $I$ thuộc trung trực của $M N$.

Ta có $M V=\frac{1}{2} B D\left(\triangle D B M\right.$ vuông tại $M$ ) và $P U=\frac{1}{2} D B$ (đường trung bình)

Suy ra $M V=P U$

Tương tự thì ta có $P V=N U$

Ta có: $\angle M V D=2 \angle M B D=2 \angle N C D=\angle N U D$ và $\angle D V P=\angle D U P$

Suy ra $\angle M V P=\angle P U N$

Xét $\triangle M V P$ và $\triangle P U N$ có $M V=P U, P V=N U, \angle M V P=\angle P U N$

$\Rightarrow \triangle M V P=\triangle P U N(\mathrm{c}-\mathrm{g}-\mathrm{c})$

Suy ra $P M=P N$. Do đó $P$ thuộc trung trực của $M N$.

Vậy $I, P, J$ thuộc trung trực $M N$ nên $I, P, J$ thẳng hàng hay $I J$ qua trung điểm $P$ của $B C$.

(d) Ta có tam giác $I M N$ cân tại $I, I J \perp M N$ nên $I T$ là đường kính của đường tròn ngoại tiếp $\triangle I M N$

Suy ra $\angle I N T=90^{\circ}$.

Suy ra $I J \cdot I T=I N^{2}$ mà $I N=I D$ suy ra $I J \cdot I T=I D^{2}$

Do đó $I D^{2}=I J \cdot I T$. Suy ra $\triangle I D J \sim \triangle I T D(\mathrm{c}-\mathrm{g}-\mathrm{c})$ nên $\angle I D J=\angle I T D$

Từ đó ta có $I D$ là tiếp tuyến của đường tròn ngoại tiếp $\triangle D T J$.

Bài 5. Đội văn nghệ của một trường THCS có 8 học sinh. Nhà trường muốn thành lập các nhóm tốp ca, mỗi nhóm gồm đúng 3 học sinh, (mỗi học sinh có thể tham gia vài nhóm tốp ca khác nhau). Biết rằng hai nhóm tốp ca bất

kỳ có chung nhau nhiều nhất là một học sinh.

(a) Chứng minh rằng không có học sinh nào tham gia từ 4 nhóm tốp ca trở lên.

(b) Có thể thành lập nhiều nhất là bao nhiêu nhóm tốp ca như vậy?

Lời giải.

(a) Giả sử có 1 học sinh tham gia 4 nhóm $A, B, C, D$ là $x$.

Khi đó $A={(x, a, b)} \quad B={(x, c, d)} \quad C={(x, e, f)} \quad D={(x, g, h)}$.

Vi các nhóm không có chung quá 1 thành viên nên các học sinh: $a, b, c, d, e, f, g, h$

là khác nhau (vô lí vì chỉ có 8 học sinh tham gia).

(b) Ta chứng minh lập được nhiều nhất là 8 nhóm.

Thật vậy, nếu có 9 nhóm, mối nhóm có 3 học sinh thì khi đó số lượt học sinh tham gia là $9 \cdot 3=27$ lượt tham gia.

Mà chỉ có 8 học sinh nên theo nguyên lý Dirichlet thì có ít nhất một học sinh có nhiều hơn hoặc bằng 4 lượt (mâu thuẫn do câu $a$ ).

(Một học sinh tham gia 1 nhóm tính là 1 lượt).

Gọi 8 học sinh là $a, b, c, d, e, f, g, h$.

8 nhóm học sinh được chia như sau:

${(a, b, c)} ; \quad{(h, b, e)} ; \quad{(b, d, f)} ; \quad{(a, d, e)} ;$

${(h, c, f)} ;  \quad{(c, e, g)} ; \quad{(a, f, g)} ; \quad{(h, d, g)} .$

 

 

 

 

 

 

 

 

 

 

 

 

 

 

 

 

 

 

 

 

 

 

 

 

 

 

 

 

 

 

 

 

 

 

 

 

 

 

 

 

 

 

 

 

 

 

 

 

 

 

 

 

 

 

 

 

 

Đề thi thử vào lớp chuyên toán Star Education năm 2021 – Lần 2

Thời gian làm bài 150 phút.

Bài 1. (2,0 diểm)
a) Tìm $m$ để phương trình $\frac{x^{2}-(3 m+1) x+2 m^{2}+2 m}{x}=0$ có hai nghiệm $x_{1}, x_{2}$ phân biệt thỏa $\left(\sqrt{x_{1}-m}+\sqrt{x_{2}-m}\right)^{4}=(2 m-1)^{2}$
b) Giải hệ phương trình $\left\{\begin{array}{l}\sqrt{x^{2}-y}=z-1 \\ \sqrt{y^{2}-z}=x-1 \\ \sqrt{z^{2}-x}=y-1\end{array}\right.$
Bài 2. (1,5 diểm) Cho các số $x, y, z$ nguyên dương thỏa $x>y>z$.
a) Cho $(x ; y ; z)$ thỏa $y z+x(x+y+z)=2021$.
Tìm giá trị nhỏ nhất của biểu thức $A=(x-y)^{2}+(x-z)^{2}+(y-z)^{2}$
b) Chứng minh rằng nếu $y$ không nhỏ hơn trung bình cộng của $x$ và $z$ thì
$$
(x+y+z)(x y+y z+x z-2) \geq 9 x y z
$$
Bài 3. (2,0 diềm) Cho $x, y$ là các số nguyên không đồng thời bằng 0 sao cho $x^{3}+y$ và $x+y^{3}$ chia hết cho $x^{2}+y^{2}$.
a) Tìm $x, y$ nếu $x y=0$.
b) Chứng minh rằng $x y \neq 0$ thì $x, y$ là nguyên tố cùng nhau.
c) Tìm tất cả cặp số nguyên $(x, y)$ thỏa đề bài.
Bài 4. (3,0 diểm) Cho tam giác $A B C$ nhọn, có trực tâm $H ; A H$ cắt $B C$ tại $D$. Trên tia đối tia $D H$ lấy điểm $M$. Đường tròn ngoại tiếp tam giác $M B H$ cắt $A B$ tại $E$ cắt $B C$ tại $K$; đường tròn ngoại tiếp tam giác $M C H$ cắt $A C$ tại $F$ và $B C$ tại $L$.
a) Chứng minh $B E F C$ nội tiếp và $\angle E M A=\angle F M A$.
b) $M E$ cắt $C H$ tại $P, M F$ cắt $B H$ tại $Q$. Chứng minh $P Q$ vuông góc $O A$ với $O$ là tâm đường tròn ngoại tiếp tam giác $A B C$.
c) $H K$ cắt $A C$ tại $U, H L$ cắt $A B$ tại $V$. Chứng minh $U V$ luôn song song với một đường thẳng cố định khi $M$ thay đổi.

Bài 5. (1,5 diểm) Trong một hội nghị Toán quốc tế có n người, mỗi người trong họ có thể nói được nhiều nhất 3 ngôn ngữ. Trong 3 người bất kì thì luôn có 2 người có thể nói chung một ngôn ngữ.
a) Cho $n \geq 9$, chứng minh răng cố một ngôn ngữ được nói bởi ít nhất 3 người.
b) Nếu $n=8$, diều kết luận của câu a) còn đúng không? Tại sao?

Đáp án có sau một tuần

 

Bài tập số học ôn thi vào lớp 10 – Phần 3

Bài 21. Chứng minh rằng với mọi số tự nhiên $n > 1$ thì $n^5 + n^4 + 1$ không là số nguyên

Lời giải

$n^5 + n^4 + 1 = n^5+n^4+n^3-n^3+1 = n^3(n^2+n+1) -(n-1)(n^2+n+1) = (n^2+n+1)(n^3-n+1)$
Mà $n^3-n+1 > 1, n^2+n+1>1$ với mọi $n>1$ nên $n^5+n^4+1$ không là số nguyên tố.

Bài 22. Tìm tất cả các số tự nhiên n sao cho ${5^{{5^{n + 1}}}} + {5^{{5^n}}} + 1$ là một số nguyên tố.

Lời giải

Đặt $m = 5^n$ ta có bài trên.

Bài 23. Tìm số nguyên tố $p$ để $p^2 + 2^p$ cũng là số nguyên tố.

Lời giải

Nhận thấy $p=3$ thỏa đề bài.
Xét $p>3$ thì $p$ lẻ và $p$ không chia hết cho 3.
Khi đó $p^2 \equiv 1 (\mod 3)$ và $2^p \equiv -1 (\mod 3)$. Do đó $p^2 + 2^p \equiv 3$ nên không là số nguyên tố.

Bài 24. Cho $p, q$ là các số nguyên tố và phương trình $x^2 – px+q=0$ có nghiệm nguyên dương. Tìm $p$ và $q$.

Lời giải

Gọi $x_1, x_2$ là nghiệm của phương trình. Ta có $x_1 + x_2 = p, x_1 x_2 = q$. Do đó $x_1, x_2 $ đều là các số nguyên dương. Giả sử $x_1 \geq x_2$.
Suy ra $x_2 = 1, x_1 = q$, $1+q = p$. Do đó $p = 3, q=2$.
Thử lại thấy thỏa đề bài.

Bài 25. Tìm tất cả các số nguyên tố $p$ sao cho tổng các ước dương của $p^4$ là một số chính phương.

Lời giải

Theo đề ta có phương trình $1+p+p^2+p^3+p^4 = x^2$.
Ta có $(2p^2+p)^2< 4x^2 < (2p^2+p+2)$.
Do đó $4x^2 = (2p^2+p+1) = 4p^2+4p^3+4p^2+4p+4$
$p^2 -2p – 3 = 0 \Leftrightarrow p=3$.

Bài 26. Tìm tất cả các số nguyên tố $p$ sao cho tồn tại các số nguyên dương $x, y$ thỏa phương trình $x(y^2-p)+y(x^2-p)=5p$.

Lời giải

$(x+y)(xy-p) = 5p$, $x+y \geq 2$ Do đó có các trường hợp sau:\\
$x+y = 5, xy-p=p$. Giải ra được $x=2, y=3, p=3$, $x=3, y=2, p=3$, $x=1, y=4, p=2$, $x=4,y=1, p=2$.\\
$x+y = p, xy -p=5$. $x^2-px+p+5 = 0$. $p^2-4(p+5) = =k^2 \Leftrightarrow (p-2)^2 – 24 = k^2 \Leftrightarrow (p-2-k)(p-2+k) = 24$. \\
Ta có $p-2-k, p-2+k$ cùng chẵn. Có các trường hợp sau:
+ $p-2-k = 2, p-2+k=12$, suy ra $p=9$ (loại)\\
+ $p-2 -k = 4, p-2+k = 6$, suy ra $p=7$. Khi đó $x+y = 7, xy = 12$. Giải ra được $x=3, y=4$ và $x=4, y=3$.

Bài 27. Cho các số nguyên dương $a, b, c, d$ thỏa $ab = cd$. Chứng minh rằng $a + b + c + d$ là hợp số.

Lời giải

Đặt $k = (a,c), a= ka’, c=kc’$, Suy ra $a’b = c’d$, suy ra $b \vdots c’$, đặt $b = mc’$, suy ra $d=ma’$.
Khi đó $a+b+c+d = ka’+mc’ + kc’+ma’ = (k+m)(a’+c’)$ là hợp số.

Bài 28. Tìm tất cả các số nguyên tố $p>q>r$ sao cho $p-r, p-q, q-r$ cũng là các số nguyên tố.

Lời giải

Nếu các số $p, q, r$ đều lẻ, thì $p-r, p-q, q-r$ đề chẵn mà là số nguyên tố và bằng 2, vô lý.
Do đó có 1 số nguyên tố chẳn, suy ra $r = 2$.
$p-2, q-2, p-q$ nguyên tố. Suy ra $p-q = 2$.
Vậy $p-2, p,p+2$ là các số nguyên tố. Suy ra $p-2=3$, $p=5$, $q=7$.

Bài 29. Tìm các số nguyên tố $p,q$ thỏa mãn hệ thức $p + q = {\left( {p – q} \right)^3}$

Lời giải

$p-q = r$ ta có $r^3 =2p+r$. Suy ra $p = \dfrac{r^3-r}{2}$ chia hết cho 3. Suy ra $p=3, q=5$.

Bài 30. Tìm tất cả các số nguyên tố $p$ sao cho hệ phương trình $p+1=2x^2,p^2+ 1=2y^2$ có nghiệm nguyên.

Lời giải

Ta xét $y, x>0$. Ta có $p = 2$ không thỏa.
$p(p-1) = 2(y-x)(y+x)$, suy ra $p |2(y-x)(y+x)$
$p|y-x$, suy ra $2(x+y)|p-1$ (vô lý)
$p|x+y$, mặt khác $p > x, p > y$, suy ra $2p>x+y$, do đó $p = x+y$. Khi đó $p-1 = 2x – 2y$. Từ đó suy ra $x = \dfrac{3p-1}{4}$, thế vào ta giải ra được $p = 7, x = 2, y = 5$.

Bài tập số học ôn thi vào 10 – Phần 2

Bài 11. Chứng minh rằng

a) Trong 5 số nguyên thì có 3 số có tổng chia hết cho 3.
b) Trong 17 số nguyên thì có 9 số có tổng chia hết cho 9.

Giải

a) Một số khi chia cho 3 có các số dư là 0, 1, 2.
Nếu trong 5 số khi chia cho 3 số có đủ 3 số dư 0, 1, 2 thì tổng 3 số này chia hết cho 3.
Nếu có 2 loại số dư thì có 3 số khi chia cho 3 có cùng một số dư, tổng của chúng chia hết cho 3.
Nếu có 1 loại số dư, thì tổng 3 số bất kì đều chia hết cho 3.
b) Đặt các số đó là $a_1, a_2, \cdots, a_{16}, a_{17}$.
Trong 5 số $a_1, \cdots, a_5$ có 3 số có tổng chia hết cho 3, không mất tính tổng quát là $a_1, a_2, a_3$. Đặt $a_1 + a_2 + a_3 = 3b_1$.
Trong 5 số $a_4, \cdots, a_8$ có 3 số có tổng chia hết cho 3, giả sử $a_4, a_5, a_6$ và đặt $a_4 + a_5+ a_6 = 3b_2$.
Tương tự ta xây dựng được các số $b_3, b_4, b_5$.
Khi đó áp dụng tiếp cho 5 số $b_1, b_2, b_3, b_4, b_5$ có 3 số có tổng chia hết cho 3, giả sử $b_1, b_2,b_3$ có tổng chia hết cho 3. Khi đó 9 số $a_1, \cdots, a_9$ có tổng chia hết cho 9.

Bài 12. (Tuyển sinh vào lớp 10 Chuyên Toán trường PTNK 2018)\ Cho $ A_n = 2018^n + 2032^n – 1964^n – 1984^n $ với $ n $ là số tự nhiên.

a) Chứng minh với mọi số tự nhiên $ n $ thì $ A_n $ chia hết cho $ 51 $.
b) Tìm tất cả những số tự nhiên $ n $ sao cho $ A_n $ chia hết cho $ 45. $

Giải

a) \item Do $ 2018 \equiv 1964 \quad \text{(mod 3)} \Rightarrow 2018^n \equiv 1964^n \quad \text{(mod 3)} . $\\
$ 2032 \equiv 1984 \quad \text{(mod 3)} \Rightarrow 2032^n \equiv 1984^n \quad \text{(mod 3)} $.\\
$ \Rightarrow A_n \ \vdots \ 3. $\\
Ta lại có $ 2018 \equiv 1984 \quad \text{(mod 17)} \Rightarrow 2018^n \equiv 1984^n \quad \text{(mod 17)} $.\\
$ 2032 \equiv 1964 \quad \text{(mod 17)} \Rightarrow 2032^n \equiv 1964^n \quad \text{(mod 17)} $.\\
$ \Rightarrow A_n \ \vdots\ 17. $\\
Do $ (3; 17) = 1 $ nên $ A_n \ \vdots \ 51 \quad \forall n$

b) Ta xét các trường hợp của $ n $ để $ A_n \ \vdots \ 5. $
Ta có $ A_n \equiv (-2)^n + 2^n -2\cdot(-1)^n $ (mod 5).
Do đó nếu $ n $ lẻ $ \Rightarrow A_n \equiv 2 \quad $(mod 5)$ \quad \text{(loại)}$.
Nếu $ n = 4k \Rightarrow A_n \equiv 2\cdot 2^{4k} -2 \equiv 2-2 \equiv 0 \quad$ (mod 5) (nhận)
Nếu $ n = 4k + 2 \Rightarrow A_n \equiv 2\cdot 2^{4k+2} -2 \equiv 8 – 2 \equiv 6$ (mod 5) (loại).
Vậy $ A_n \ \vdots \ 5 \Leftrightarrow n \ \vdots \ 4. $
\item Ta xét các trường hợp của $ n $ để $ A_n \ \vdots \ 9. $
Ta có \begin{align*}
A_n &\equiv 2^n + (-2)^n – 2^n – 4^n \quad \text { (mod 9)}\\\\
&\equiv 2^n -4^n \quad \text { (mod 9) \quad (Do n chẵn).} \\\\
& \equiv 2^n(1-2^n) \quad \text { (mod 9)}
\end{align*}
Vì $ (2;9 ) = 1 \Rightarrow 2^n – 1 \ \vdots \ 9$.
Xét $ n= 3k $ với $ k \in \mathbb{N} $. Ta có $ A_n \equiv 2^{3k} – 1 \equiv (-1)^k – 1 \quad \text { (mod 9)} \Rightarrow k$ chẵn
Xét $ n= 3k + 1 $ với $ k \in \mathbb{N} $. Ta có $ A_n \equiv 2^{3k + 1} – 1 \equiv 2\cdot(-1)^k – 1 \quad \text { (mod 9) \quad (loại)}. $
Xét $ n= 3k + 2 $ với $ k \in \mathbb{N} $. Ta có $ A_n \equiv 2^{3k + 2} – 1 \equiv 4\cdot(-1)^k – 1 \quad \text { (mod 9) \quad (loại)}. $

Vậy $ A_n \ \vdots \ 45 \Leftrightarrow n \ \vdots \ 12. $

Bài 13. Tìm các nghiệm nguyên không âm $(x, y)$ của phương trình
${\left( {xy – 1} \right)^2} = {x^2} + {y^2}$

Giải

$(xy-6)^2 – (x+y)^2 = -13$.
$(xy-6-x-y)(xy-6+x+y) = -13$.
Ta có $xy – 6 +x+y \leq xy – 6 -x-y$ nên có các trường hợp.
$xy -6 -x-y = -13, xy -6 +x+y = 1$, giải ra được $(x;y)$ là $(7;0), (0;7)$;
$xy – 6 -x-y=-1, xy-6+x+y = 13$ (VN);
$Vậy phương trình có nghiệm $(0;7), (7;0)$.

Bài 14. Chứng minh rằng phương trình ${y^2} + y = x + {x^2} + {x^3}$ không có nghiệm nguyên dương.

Giải

Ta có $x^3 = (y-x)(y+x+1)$.
Gọi $d$ là ước nguyên tố chung lớn nhất của $y-x, y+x+1$, nếu $d$ là số nguyên tố thì $d|x, d|y$, suy ra $d|1$ (vô lý), Vậy $y-x, y+x+1$ nguyên tố cùng nhau.
Do đó $y -x = a^3, y+x+1 = b^3, ab=x$.
Ta có phương trình $b^3-a^3 = 2ab+1$ với $a, b$ nguyên dương và $b > a\geq 1$. Ta có $b^3-a^3 \geq a^2+b^2+ab > 2ab + 1$.
Vậy phương trình không có nghiệm trong tập các số nguyên dương.

Bài 15. Tìm tất cả các bộ ba số nguyên dương thỏa phương trình:
${\left( {x + y} \right)^2} + 3x + y + 1 = {z^2}$

Giải

Ta có $(x+y)^2 < z^2 < (x+y+2)^2$. Do đó $z^2 = (x+y+1)^2$ hay $(x+y+1)^2 = (x+y)^2+3x+y + 1 \Leftrightarrow y = x$.
\Vậy bộ nghiệm là $(n, n, 2n+1)$ với $n$ là số nguyên dương.

Bài 16. Tìm nghiệm nguyên dương của phương trình sau
$xy + yz + zx – xyz = 2$

Giải

Vai trò của $(x, y, z)$ là như nhau, giả sử $x \geq y \geq z$.
$\dfrac{1}{x}+\dfrac{1}{y}+\dfrac{1}{z} – 1 = \dfrac{2}{xyz} > 0$. Suy ra $\dfrac{3}{z} -1 > 0$, suy ra $z < 3$.
Nếu $z = 1$ thì $x+y = 2$ ta có $x = y = 1$.
Nếu $z=2$ thì $2(x+y)-xy = 2 \Leftrightarrow (x-2)(y-2) = 2$, giải ra được $x = 4, y = 3$.
Do tính đối xứng nên nghiệm của phương trình là $(1, 1, 1), (4,3,2)$ và các hoán vị.

Bài 17. Tìm tất cả các số tự nhiên x, y thỏa: ${5^x} = {y^4} + 4y + 1$

Giải

Có một nghiệm là $(0;0)$.
Dễ thấy $y$ chẵn nên $y^4+4y+1 \equiv 1 (\mod 8)$. Suy ra $x$ chẵn, $x = 2k$. Khi đó $(5^k)^2 = y^4 + 4y+1$ là số chính phương.
Ta có $y\geq 1$ nên $y^4 < y^4+4y + 1 < (y^2+2)^2$. Suy ra $y^4+4y + 1 = (y^2+1)^2 \Leftrightarrow y = 2$, suy ra $x = 2$.
Vậy có 2 cặp nghiệm $(0;0), (2;2)$.

Bài 18. Giải phương trình nghiệm tự nhiên $x – {y^4} = 4$ với $x$ là số nguyên tố.

Giải

$x = y^4+4 = (y^2-2y+2)(y^2+2y+2)$ là số nguyên tố khi và chỉ khi $y^2-2y + 2 = 1$ hay $y=1$. Từ đó $x=1$.

Bài 19. Tìm nghiệm nguyên của phương trình sau
${\left( {{x^2} – {y^2}} \right)^2} = 1 + 16y$

Giải

Dễ thấy nghiệm là $(-1;0), (1;0)$.
Ta có $y \geq 0$, vì $x$ thỏa pt thì $-x$ cũng thỏa nên có thể giả sử $x\geq 0$.
Ta có $(x^2-y^2)^2 = 1 + 16y >1$, suy ra $x^2 > y^2 \Rightarrow x \geq y + 1$.
Nếu $x \geq y + 2$, suy ra $x^2-y^2 \geq 4y + 4 \Rightarrow (x^2-y^2)^2 > 1+16y$.
Do đó $x = y + 1$, suy ra $(1+2y)^2 = 1+16y \Leftrightarrow 4y^2 – 12y = 0 \Leftrightarrow y = 3$. Suy ra $x = 4$.
Vậy nghiệm là $(-4;3), (4;3),(-1;0), (1;0)$.

Bài 20. Chứng minh rằng với mọi số tự nhiên $n > 1$ thì $n^5 + n^4 + 1$ không là số nguyên tố.

Giải

$n^5 + n^4 + 1 = n^5+n^4+n^3-n^3+1 = n^3(n^2+n+1) -(n-1)(n^2+n+1) = (n^2+n+1)(n^3-n+1)$
Mà $n^3-n+1 > 1, n^2+n+1>1$ với mọi $n>1$ nên $n^5+n^4+1$ không là số nguyên tố.

Giải bài toán bằng đại lượng cực biên – Phần 2

(Bài viết dành cho các em học sinh lớp 8, 9, 10)

Ví dụ 1. Tìm $n$ lớn nhất sao cho tồn tại $n$ điểm mà 3 điểm bất kì đều tạo thành tam giác vuông.

Lời giải. 

Ta thấy $n=3, n=4$ đều tồn tại. Ta chứng minh $n\geq 5$ thì không tồn tại. \
Giả sử ngược lại, tồn tại 5 điểm, sao cho 3 điểm bất kì đều tạo thành tam giác vuông. Khi đó ta chọn hai điểm sao cho có độ dài lớn nhất. Khi đó các điểm còn lại đều nằm trên đường tròn đường kính là đoạn thẳng này. Khi đó 3 điểm thuộc 2 nửa đường tròn, khi đó có ít nhất 2 điểm cùng thuộc một nửa, từ đó tồn tại một tam giác khác vuông có đỉnh là 2 điểm này cùng một điểm thuộc đường kính. Do đó không thỏa đề bài.

Nhận xét. Đây là một bài toán cực trị dạng tìm số nhỏ nhất, lớn nhất của n để thỏa điều kiện nào đó. Những kiểu bài tập này thường ta cứ xét các trường hợp nhỏ và cố gắng xây dựng cấu hình thỏa, đối với bài này cấu hình rất dễ tìm, với trường hợp $ n = 5$, để chứng minh không tồn tại, ta sử dụng cực biên, kết hợp với phản chứng để cho lời giải trọn vẹn, chọn độ dài lớn nhất giúp mình gôm hết các điểm vào thành một đường tròn, từ đó giúp giải được bài toán.

Ví dụ 2. Trên một mặt bàn đặt một số các đồng xu với kích cỡ không giống nhau đôi một (các đồng xu không được đè lên nhau và phải nằm sấp hoặc ngửa trên bàn). Chứng minh rằng dù ta đặt như thế nào đi nữa, cũng luôn tồn tại một đồng xu chỉ tiếp xúc được với nhiều nhất 5 đồng xu khác.

Lời giải. Đồng xu càng to thì nhiều đồng xu có thể tiếp xúc với nó, còn ngược lại thì càng nhỏ, do đó để càng ít đường tròn tiếp xúc nó, ta chọn đồng xu nhỏ nhất.

Chọn đồng xu có bán kính nhỏ nhất, thì đồng xu này chỉ tiếp xúc không quá 5 đồng xu khác. Giả sử nó có thể tiếp xúc với 6 đồng xu khác. Khi đó $A$ là tâm đường tròn, tâm các đường tròn còn lại là $A_1, \cdots, A_6$. Khi đó tồn tại $A_iA_{i+1} \leq 60^\circ$, suy ra $A_iA_{i+1} < AA_i$ vô lý, vì bán kính của $(A)$ là nhỏ nhất.

Ví dụ 3. Cho $n$ điểm trong mặt phẳng biết rằng cứ 3 điểm bất kì tạo thành một tam giác có diện tích không lớn hơn 1. Chứng minh rằng $n$ điểm thuộc một hình tam giác có diện tích không lớn hơn 4.

Lời giải. Gọi $A, B, C$ là 3 điểm tạo thành tam giác sao cho $ABC$ có diện tích lớn nhất. Từ $A, B, C$ vẽ các đường song song với các cạnh đối diện, các đường thẳng cắt nhau tại $A’, B’, C’$ ta chứng minh các điểm thuộc cạnh hoặc miền trong tam giác $A’B’C’$. \
Thật vậy, nếu có điểm nào nằm ngoài tam giác $A’B’C’$ thì điểm đó kết hợp với hai trong 3 điểm $A, B, C$ sẽ có diện tích lớn hơn diện tích tam giác $ABC$, vô lý. \
Do $S_{A’B’C’} = 4S_{ABC} \leq 4$.

Ví dụ 4. (Sylvester) Trong mặt phẳng cho $n$ điểm phân biệt, sao cho mỗi đường thẳng đi qua hai điểm thì đi qua ít nhất một điểm khác. Chứng minh rằng $n$ điểm này cùng thuộc một đường thẳng.

Lời giải. Giả sử không phải tất cả các điểm cùng thuộc một đường thẳng. Khi đó ta xét khoảng cách từ một điểm đến đường thẳng qua ít nhất 3 điểm, trong các khoảng cách này có khoảng cách nhỏ nhất. Giả sử $P$ là điểm có khoảng cách từ $P$ đến $d$ là nhỏ nhất, với $d$ là đường thẳng qua các điểm $A, B, C$ theo thứ tự. \
Gọi $H$ là hình chiếu của $P$ trên $d$, $D, E$ là hình chiếu của $A, B$ trên $B$ trên $PA, PC$. Nếu $H$ thuộc tia $BA$ thì $BE < PH$, nếu $H$ thuộc đoạn $BC$ thì $BD < PH$. Mâu thuẫn với $PH$ là nhỏ nhất. \
Vậy tất cả các điểm cùng thuộc một đường thẳng.

Việc chọn phần tử lớn nhất, nhỏ nhất thể hiện ưu thế của của các phần tử đó so với các đối tượng khác, đó chưa chắc là cái thỏa, nhưng cũng cũng có ưu thế hơn, giống khi xét tuyển, các thí sinh có điểm trung bình cao chưa chắc là giỏi nhất, nhưng là những người có ưu thế hơn điểm thấp, khi chọn trong nhóm đó sẽ tìm được nhiều người giỏi hơn là chọn trong nhóm thấp điểm, do đó vượt trội một khía cạnh nào tính ra là một lợi thế để so sánh.

Ta tiếp tục với việc chứng minh các bài toán về tồn tại các đối tượng thỏa yêu cầu nào đó.

Ví dụ 5. Cho 3 trường, mỗi trường có $n$ học sinh, biết rằng cứ mỗi học sinh thì quen ít nhất $n + 1$ học sinh của hai trường khác. Chứng minh rằng có thể chọn được từ mỗi trường một bạn sao cho 3 bạn này đôi một quen nhau.

Lời giải. Giả sử 3 trường là $X, Y, Z$. Tồn tại một người có số người quen ở cùng một trường khác là nhiều nhất, giả sử $A$ thuộc $X$ có số người quen ở trường $Y$ nhiều nhất là $k$. Khi đó số người quen của $A$ ở $Z$ ít nhất là $n+1-k$. Nếu nhóm người quen $A$ ở $Z$ quen với số người quen $A$ ở $X$ có hai người quen nhau thì ta có điều chứng minh.\
Ngược lại xét người quen $A$ ở $Z$, đặt là $B$ quen số người ở $Y$ tối đa là $n-k$, khi đó $B$ quen ở $X$ ít nhất là $n+1 – (n-k) = k+1$, mâu thuẫn với cách chọn $A$. (Mâu thuẫn).

Ví dụ 6. Một bảng $2n \times 2n$ ô, người ta đánh dấu bất kì $3n$ ô trong bảng. Chứng minh rằng tồn tại $n$ dòng và $n$ cột sao cho $3n$ ô được đánh dấu thuộc $n$ dòng và $n$ cột này.

Lời giải. Chọn $n$ dòng sao cho số ô được tô là lớn nhất, ta chứng minh rằng số ô được tô trong $n$ dòng này là không ít hơn $2n$ ô.
Thực vậy giả sử số ô được tô là ít hơn $2n$, khi đó $n$ dòng còn lại có nhiều hơn $n$ ô được tô, nên có ít nhất một một dòng có 2 ô được tô.

Do đó $n$ dòng đã chọn, mỗi dòng ít nhất 2 ô được tô nên tổng số ô hơn hoặc bằng $2n$ (mâu thuẫn).
Vậy ta chỉ cần chọn $n$ cột chứa các ô được tô màu nhưng chưa được chọn trong $n$ dòng trên thì sẽ có điều cần chứng minh.

Ví dụ 7. Một bữa tiệc có 10 học sinh tham gia, biết rằng mỗi học sinh quen với ít nhất là 5 người. Chứng minh rằng có thể sắp xếp 10 học sinh ngồi vào một bàn tròn sao cho hai người kế nhau thì quen nhau.

Lời giải. Giả sử chuỗi người quen dài nhất có độ dài là $k$, $A_1A_2…A_k$, ta thấy các người còn lại không ai quen $A_1, A_k$ nên suy ra $k \geq 6$. \
Nếu $k = 6$, suy ra $A_1$ và $A_6$ quen nhau, khi đó trong các người còn lại $A_7$ quen một trong cái người giả sử là $A_i$, khi đó ta có chuỗi $A_7A_iA_{i-1}A_1A_6A_{i+1}$ có độ dài hơn 6, vô lý.\
Nếu $k =7$, khi đó $A_1$ quen từ $A_2$ đến $A_6$ và $A_7$ quen $A_2$ tới $A_6$, khi đó có một vòng $A_2A_7A_6A_5A_4A_3A_1A_2$. Khi đó sẽ có một người trong nhóm còn lại thì ta sẽ có chuỗi dài hơn, mâu thuẫn.\
Nếu $k=8,9$ xét tương tự, ta sẽ có $k=10$. Giả sử có chuỗi $A_1\cdots A_{10}$. Khi đó tồn tại $k>i$ sao cho $A_1$ quen $A_k$ và $A_{10}$ quen $A_i$, khi đó có cách xếp thỏa đề bài là $A_1A_k\cdot A_iA_{10}A_9…A_k$.

Bài tập Bài tập nguyên lý cực biên

Tài liệu tham khảo. 

  1. Problems – Solving Stretagies – Arthur Hegel
  2. Giải bài toàn bằng đại lượng cực biên – Nguyễn Hữu Điển

Nguyên lý cực biên – Phần 1

(Bài viết dành cho học sinh lớp 8,9 và đầu lớp 10)

Có một câu chuyện thú vị thường thấy là trong lớp học những người nào ngồi bàn đầu hay bàn cuối thì thường hay bị gọi lên bảng trả bài hơn là những người khác, vì sao như vậy? Thực sự vì hai vị trí đó là vị trí đầu và cuối, tức là vị trí biên, vị trí “đặc biệt” hơn các vị trí khác, nên dễ được chú ý hơn.

Hoặc có một bài toán đơn giản sau: Tam giác $ABC$, $M$ thuộc cạnh $BC$, với vị trí nào của $M$ thì $AM$ đạt giá trị lớn nhất? (nhỏ nhất?). Dễ nhận ra rằng $AM \leq AB$ hoặc $AM \leq AC$, do đó $AM$ lớn nhất chỉ khi $M$ là một trong hai vị trí $B$ hoặc $C$, đó chính là vị trí biên của đoạn thẳng.

Do đó các vị trí biên của một tập hợp $X$ nào đó luôn có những đặc điểm mà vị trí khác không có được, kiểu nếu lệch ra một tí thì “bay màu” khỏi $X$.

Nguyên lý cực biên cũng như nguyên lý quy nạp, đó là một trong các nguyên lý quan trọng để chứng minh các định lý hay các bài toán. Xuất phát tự quan hệ thứ tự trong tập các số thực, và tiên đề xây dựng số tự nhiên, ta có các tính chất sau

  • Mọi tập con khác rỗng hữu hạn của tập số thực luôn có phần tử lớn nhất và nhỏ nhất.
  • Mọi tập con khác rỗng của tập các số tự nhiên đều có phần tử nhỏ nhất
  • Mọi tập con khác rỗng bị chặn trên của tập số nguyên có phần tử lớn nhất, bị chặn dưới thì có phần tử nhỏ nhất.

Nguyên lý cực biên xuất hiện nhiều trong các chứng minh, trong bài viết nhỏ này tôi chỉ giới thiệu một số bài toán cơ bản thường gặp để giúp các em học sinh nắm được kĩ thuật chứng minh này, từ đó vận dụng để làm các bài toán khó hơn.

Việc sử dụng nguyên lí cực hạn có cái quan trọng nhất là mình sử dụng đặc điểm đặc biệt của đại lượng cực biên, xem như một giả thiết mới để khai thác, kết hợp với các kĩ thuật sắp xếp, phản chứng để giải quyết bài toán.

Ta xét vài ví dụ sau

Bài 1. Cho số thực $x$ chứng minh rằng tồn tại duy nhất số nguyên $n$ sao cho $n\leq x < n+1$. ($n$ được gọi là phần nguyên của $x$, kí hiệu là $[x]$.

Lời giải. 

Nhận xét: rõ ràng $n$ là số nguyên mà nhỏ hơn và “gần” $x$ nhất, tức là nếu $n$ tăng thêm một đơn vị thì nó sẽ vượt qua $x$. Từ ý đó ta có thể giải như sau:

Đặt $A = \{n \in \mathbb{Z}, n \leq x \}$, ta thấy $A$ là tập con khác rỗng của $\mathbb{Z}$, bị chặn trên bởi $x$ nên tồn tại phần tử lớn nhất, đặt là $n_\circ$. Ta chứng minh $n_\circ \leq x < n_\circ+1$.

Rõ ràng $n_\circ \in A$ nên $n_\circ \leq x$.

Giả sử $n_\circ + 1 \leq x$ thì $n_\circ \in A$ và $n_\circ + 1  > n_\circ $ vô lí vì $n_\circ$ là phần tử lớn nhất của $A$. Do đó $n_\circ +1 > x$

Từ đó ta có $n_\circ \leq x < n_\circ + 1$.

Bước kế tiếp là chứng minh duy nhất,giả sử tồn tại $n’$ nguyên thỏa $n’\leq x < n’+1$. \

Nếu $n’ > n_\circ$ thì $n’ \geq n_\circ+1 > x$, vô lí, tương tự với $n_\circ > n’$.

Do đó $n’ = n_\circ$.

Bài 2. Cho hai số nguyên dương $a, b$. Chứng minh rằng tồn tại duy nhất cặp số $q, r$ sao cho $0 \leq r \leq b-1$ và $$a = bq + r$$

Lời giải. Do $0 \leq r \leq b-1$ nên mình thấy rằng, $q$ trong đẳng thức trên là số lớn nhất để hiệu $a-bq$ không không âm.

Đặt $A = \{a-bq \leq 0, q\in \mathbb{N} \}$.

Rõ ràng $A$ khác rỗng vì $a-b \cdot 0 > 0$, và là tập con của tập các số tự nhiên. Khi đó $A$ có phần tử nhỏ nhất, đặt là $r$, ta có $q$ để $r = a-bq$. Ta chứng minh $0 \leq r \leq b-1$.

Rõ ràng $r \in A$ nên $r \geq 0$.

Ở ý còn lại, ta giả sử $r \geq b$, khi đó $r-b = a-bq-b = a-b(q+1) \geq 0$ và $r-b < r$, do đó $r-b$ thuộc $A$ và nhỏ hơn $r$,  mâu thuẫn với $r$ là số nhỏ nhất thuộc $A$.

Giả sử tồn tại cặp $q’, r’$ thỏa đề bài. Khi đó $a = bq+r = bq’+r’$

suy ra $r-r’ = b(q’-q)$ chia hết cho $b$ mà $|r-r’| \leq b-1$, do đó $r-r’=0$, và $q-q’=0$. Ta có điều cần chứng minh.

Ví dụ 3. Cho $a, b$ là hai số nguyên dương, gọi $d$ là ước chung lớn nhất của $a$ và $b$. Chứng minh rằng tồn tại các số nguyên $x, y$ thỏa $$d = x\cdot a + y \cdot b$$

Lời giải. Ý tưởng tương tự như bài trên, xét tập các tổ hợp tuyến tính dương của $a, b$ có dạng $xa + yb$,

Đặt T = ${xa + yb| x,y \in Z, xa +yb >0}$. Rõ ràng $T$ khác rỗng và là tập con của tập các số tự nhiên nên có phần tử nhỏ nhất, đặt là $e$.
Khi đó T có phần tử nhỏ nhất, ta đặt $e = xa + yb$.
Giả sử $a = ek +r$, với $ 0 \leq r < e$ , suy ra $r = a – ek = a – (xa +yb).k = a(1 – xk) + b. yk$.

  • Nếu $r >0$ thì $r \leq e$ mâu thuẫn vì $e$ là phần tử nhỏ nhất của $T$.
  • Vậy $r =0$ suy ra $e|a$. Chứng minh tương tự ta có $e|b$ do đó $e|d$.
  • Mặt khác $d|a, d|b$ suy ra $d|(xa + yb)$ hay $d|e$. Từ đó ta có $d = e$.

Ví dụ 4. Chứng minh rằng $\sqrt{2}$ là số vô tỉ.

Lời giải. Việc chứng minh $\sqrt{2}$ là số vô tỉ có nhiều cách, nhìn chung đều sử dụng phản chứng, và tính chất số học, lần này ta trình bày với phản chứng kết hợp với đại lượng cực biên.

Giả sử $\sqrt{2}$ không là số vô tỉ, tức là $\sqrt{2} = \dfrac{a}{b}$ trong đó $a, b$ là các số nguyên dương, suy ra $b\sqrt{2} = a$ là số nguyên dương.

Đặt $A = \{n| n, n\sqrt{2} \in \mathbb{N}\}$. Rõ ràng, $A$ khác rỗng là con của tập các số nguyên dương, nên có phần tử nhỏ nhất, đặt là $k$.

Ta có $k, k\sqrt{2}$ nguyên dương, suy ra $k(\sqrt{2}-1)$ nguyên dương.

Và $k(\sqrt{2}-1)\sqrt{2} = 2k – k\sqrt{2}$ cũng nguyên dương.

Do đó $k(\sqrt{2}-1)$ thuộc $A$ và $0 < k(\sqrt{2}-1) < k$ vô lí vì $k$ là nhỏ nhất.

Ví dụ 5. Chứng minh rằng không tồn tại các số nguyên dương $x, y, z, t$ sao cho $$x^2+y^2=3(z^2+t^2)$$

Lời giải. Giả sử tồn tại bộ 3 số nguyên dương thỏa đề bài, ta chọn bộ thỏa $x^2+y^2$ nhỏ nhất. Khi đó $x^2+y^2$ chia hết cho 3, suy ra $x, y$ đều chia hết cho $3$, khi đó $x= 3x’, y=3y’$, suy ra $z^2+t^2 = 3(x’^2+y’^2)$, thì bộ $(z,t,x’,y’)$ cũng thỏa đề bài, nhưng $z^2 +t^2 < x^2+y^2$. Mâu thuẫn.

Do đó phương trình không có nghiệm trong tập các số nguyên dương.

(Hết phần 1)

Tài liệu tham khảo. 

[1] Giải toán bằng phương pháp Đại lượng cực biên – Nguyễn Hữu Điển

[2] Problems Solving Strategies –